Cases 3
Cases 3
Week 1 Day 1
2) OCA is the most common form of albinism and is inherited in an autosomal recessive pattern.
Another less common form of albinism is ocular albinism (OA), in which the disease is isolated to
the eye, caused by gene mutations with X-linked inheritance pattern.
Syndromic forms of albinism include Chediak-Higashi syndrome, which has an autosomal recessive
pattern of inheritance.
Because there is no treatment for albinism, early diagnosis is essential to prevent complications,
the most significant of which include visual impairments and skin cancer.
3) Impaired melanin biosynthesis in melanocytes → absence or ↓ melanin → hypopigmentation of skin,
hair, and eyes.
Depending on the OCA type, there may be complete absence of pigmentation or a variable amount of
melanin production.
The number of melanocytes is not reduced.
Melanin is also important for the development of structures of the eye and the routing of nerve fibers
from the retina to the optic chiasm → ↓ vision in albinism.
A 33-year-old man presents to his primary care provider because of white spots on both
hands. The white spots have expanded in the last few months; they are not tender nor
ulcerated. His past medical history is relevant for hypothyroidism. Upon physical
examination, the patient shows depigmented macules on both hands, back, and
shoulders. Under a Wood lamp, the lesions emit a bright fluorescence and appear sharply
demarcated. There are no signs of inflammation in the skin lesions. His vital signs are
within normal limits.
Generalized vitiligo is the most common presentation. Vitiligo is characterized by the typically bilateral
distribution of depigmented patches and is usually not associated with other symptoms. Depigmented
patches can appear anywhere on the body but most commonly are seen on the hands, face, and genitals.
Woods lamp examination reveals depigmented patches, and it is the diagnostic test for vitiligo.
The diagnosis of vitiligo is clinical, with histological examination of skin biopsies reserved for
complicated cases in which the diagnosis is uncertain. The clinical manifestations and characteristics of
vitiligo include asymptomatic well-defined depigmented macules and patches that fluoresce under
Wood lamp examination. There is no inflammation in the vitiligo-affected skin areas, and depigmented
hairs are often seen within the skin lesions.
The treatment mostly consists of sun protection, topical steroids, or topical calcineurin inhibitors.
Corticosteroids act via various genomic and non-genomic pathways such as transactivation, transrepression, histone medication and Src kinase signaling.
2) Tinea versicolor (also known as pityriasis versicolor) is one of the most common superficial
cutaneous fungal infections. It is characterized by macules and patches that can be hypopigmented,
hyper-pigmentated, or erythematous. These macules are typically distributed around the trunk and
proximal upper extremities. Tinea versicolor is caused by saprophytic, lipid-dependent
yeast Malassezia. Tinea versicolor macules emit fluorescence under a Wood lamp examination and
show a characteristic fine scaling. Treatment is with topical antifungals.
Tinea corporis refers to a dermatophyte skin infection other than the face, feet, groin, scalp, or beard
hair. The causative organism includes the filamentous fungi from Trichophyton, Microsporum,
and Epidermophyton. Tinea corporis is characterized by pruritic, circular, erythematous, and scaling
plaques. These plaques spread centrifugally and present a central clearing and a clear raised border,
resulting in an annular plaque.
A 69-year-old man presents to his provider with an enlarging, ulcerative pink plaque on
his face. It has been present for 5 weeks and is shown in the image below. Physical
examination reveals a friable lesion that bleeds easily. Medical history is remarkable
for type 1 diabetes mellitus complicated by end-stage kidney disease, which required
kidney transplantation 5 years ago. The patient also reports a history of common viral
warts but has not had any in several years. A skin biopsy of the lesion reveals full-
thickness keratinocyte atypia with keratin pearls.
typically ulcerate.
Actinic keratoses are premalignant lesions that can progress to cutaneous squamous cell carcinoma.
The first line of treatment involves standard excision; however, when this strategy fails to remove the
entire breadth of the tumor, the immediate indication is to proceed with a follow-up resection using
Mohs surgery. During this surgery, skin layers that contain cancerous tissue are progressively
removed and examined until it is confirmed that only cancer-free tissue is present. In cases where
further surgery is not possible, the indication is to proceed with radiation therapy.
American Joint Commission on Cancer 2018 TNM system:
Only applicable to cSCC of the head and neck area (lip, ear, face, scalp, and neck).
The increase in size and irregular borders in this man’s lesion warrants a biopsy.
A lesion’s diameter can suggest the presence of a possible malignant lesion. A diameter > 6 mm is
suggestive of a possible malignancy. However, this is not a prognostic factor. In fact, many lesions that have
a radial growth phase, such as superficial spreading melanoma, have a better prognosis. Determining the
diameter of the lesion can aid in the diagnosis of the disease but has no effect on the outcome.
The S-100 protein is a tumor marker that is used to follow the treatment response and possible recurrence
of a number of cancers, including melanoma. This tumor marker is not specific for melanoma and should
not be used for diagnosis, however. Levels of S-100 do not correspond to prognosis; however, changes in
levels can suggest a response to treatment.
The evolution of a lesion over time (changes in its size or color) is another way to aid in the diagnosis of
melanoma. These changes can be considered normal and on their own are not indicative of disease or
prognosis. However, observing these kinds of changes can aid in the diagnosis of melanoma.
Melanoma is associated with mutations in the BRAF kinase gene. These mutations can be inherited or
spontaneous. Other genetic factors associated with melanoma play a lesser role. While a BRAF gene
mutation may increase a patient’s likelihood of developing melanoma, it is not indicative of the progress or
prognosis of the disease.
5) Wide local excision
Mohs micrographic surgery (used for in situ melanoma and lentigo maligna of the head and neck)
Regional lymph node dissection
Surgical metastasectomy (for isolated distant metastases)
For stage III disease (lymph node involvement) and stage IV (metastatic disease):
Immunotherapy (combination of):
Nivolumab or pembrolizumab (anti–programmed cell death antibody)
Ipilimumab (monoclonal antibody to cytotoxic T lymphocyte–associated antigen 4)
Targeted therapy:
For BRAF V600E/K mutation: combination vemurafenib (BRAF kinase inhibitors) with cobimetinib
(MEK inhibitor)
Other option for BRAF V600E/K mutation: dabrafenib + trametinib, encorafenib + binimetinib
For KIT mutation: imatinib (tyrosine kinase inhibitor)
Radiotherapy:
Mostly for palliation in locally advanced disease
Skeletal Structure and Function
Week 1 Day 2
Examples:
• Nevi
• Warts
• Lichen planus
• Insect bites
• Seborrheic keratoses
• Molluscum contagiosum
• Angiomas
• Skin cancers
• Plaque
• Raised skin lesion measuring > 1 cm in
diameter
• Usually have palpable surface change as
lesion arises from the epidermis
• May have a flat-topped or rounded
appearance.
Adherens Junctions:
Cardiac Dysfunction in Dilated Cardiomyopathy.
Adherens junctions play a role in maintaining the structural integrity of cardiac myocytes. Disruption of adherens
junctions, often due to genetic factors or inflammation, can lead to dilated cardiomyopathy and impaired cardiac
function.
Desmosomes:
Pemphigus Vulgaris.
Pemphigus Vulgaris is an autoimmune blistering disorder where autoantibodies target desmogleins, which are
components of desmosomes in the skin. Disruption of desmosomes results in loss of cell adhesion, leading to the
formation of intraepidermal blisters.
Gap Junctions:
Arrhythmias in the Heart.
Gap junctions facilitate direct communication between adjacent cells by allowing the passage of ions and small
molecules. In the heart, disruption of gap junctions can lead to impaired electrical coupling between cardiac cells,
contributing to arrhythmias and conduction abnormalities.
·
Hemidesmosomes
Bullous Pemphigoid.
Bullous Pemphigoid is an autoimmune blistering disorder where autoantibodies target components of hemidesmosomes,
particularly BP180. Disruption of hemidesmosomes results in separation of the epidermis from the underlying basement
membrane, leading to the formation of tense blisters.
Skeletal Structure and Function
Week 1 Day 3
1) Define and discuss the skin infectious disorders with bacterial nature.
2) Differentiate various dermatologic descriptors in infectious skin disorders.
3) Discuss pathology and pathophysiology aspects of infectious skin disorders.
&
A 74-year-old woman presents to the clinic for evaluation of an erythematous and
edematous skin rash on her right leg that has progressively worsened over the last 2
weeks. Her medical history is significant for hypertension and type 2 diabetes
mellitus. She takes lisinopril and metformin. Her vital signs are blood pressure
152/92 mm Hg, heart rate 76/min, respiratory rate 12/min, and temperature 37.8°C
(100.1°F). On physical exam, the patient appears alert and oriented. Observation of
the lesion reveals a -
Ipoorly demarcated region of erythema and edema along the
anterior aspect of the right tibia. Within the region of erythema is a 2- to 3-mm linear
break in the skin with no serous or purulent discharge. The lesion is tender and warm
when palpated. No vesicles, pustules, papules, or nodules are present. Ultrasound of
the lower extremity is negative for deep vein thrombosis and skin abscess. Blood
cultures are pending.
1) What is the most likely diagnosis based on history and physical examination?
2) What are the predisposing factors for this disease? What microorganisms could
be the cause?
3) What are the possible treatment options?
4) What diseases should be considered for differential diagnosis?
1) The clinical scenario describes an elderly diabetic woman presenting with an erythematous,
edematous, poorly demarcated skin lesion with pain and warmth on palpation. Middle-aged adults
and the elderly present with cellulitis (infection at the level of the deep dermis) more commonly than
younger individuals. The clinical manifestations of erythema, edema, pain, and warmth with a poorly
demarcated border aid to make the diagnosis.
2) Various factors predispose an individual to cellulitis, including disturbances in the skin barrier
secondary to trauma, skin inflammation (eczema, radiation), edema secondary to venous
insufficiency or lymphatic blockage, obesity, diabetes, HIV, skin breaks between toes, and
preexisting skin infections.
Streptococcus pyogenes (nonpurulent cellulitis) and Staphylococcus aureus (purulent cellulitis)
are frequent causes of soft tissue skin infections, but in a minority of cases, even gram-negative
bacilli may cause cellulitis.
·
3) Antibiotics such as cephalexin, dicloxacillin, and clindamycin are the options for treatment.
If methicillin-resistant S. aureus is suspected, amoxicillin plus trimethoprim-sulfamethoxazole is a
useful management regimen.
If treatment is unsuccessful, consider admitting the patient to the hospital for parenteral therapy.
4) Erysipelas frequently occurs more often in children and the elderly. In most cases beta-hemolytic
streptococci serve as the pathogenic agent in the skin infection. The clinical features of warmth,
erythema, and edema may appear similar to cellulitis, but there are notable differences: acute onset,
clear demarcation of erythema (plaque-like raised fiery red skin lesions), fever and/or chills, face
(butterfly shape) and ear (Milian ear sign) involvement. Penicillin and amoxicillin are the first-line
medications for the outpatient management of erysipelas.
Contact dermatitis can be caused by many irritants, including wood, fiberglass, plants, paper, dust,
water, detergents, solvents, and acidic and alkaline items. Irritant contact dermatitis consistently
manifests as erythematous, edematous, and vesicular lesions that may develop bullae and ooze.
Avoidance of irritants and, if necessary, treatment with topical corticosteroids are effective strategies
for managing irritant contact dermatitis.
Clostridial myonecrosis, known as gas gangrene, involves once healthy muscle being gradually
invaded and destroyed by Clostridium perfringens (traumatic gas gangrene), Clostridium septicum
(spontaneous gas gangrene), and, less commonly, other Clostridium species. The clinical diagnosis is
based on the observation of pain, systemic signs, and crepitus in the soft tissue. Traumatic and
spontaneous gas gangrene should be surgically debrided and the patient should be treated with
antibiotics.
A 58-year-old woman with a history of chronic alcohol abuse presents to the
emergency department because of leg pain and the skin changes seen in the image
below. She had an intramuscular injection in the same leg 2 days ago, and her
symptoms began last night. She has had diabetes mellitus for 13 years. Her
medications include metformin and insulin. She appears confused. The blood
pressure is 90/70 mm Hg, pulse 115/min, respiratory rate 21/min, and temperature
39.5℃ (103.1℉). The serum creatinine is 2.5 mg/dL.
Brawny edema, bullae, and black discoloration of the skin may follow as the infection spreads rapidly
through the lymphatic system and blood vessels into deeper layers of tissue.
Renal failure followed by shock and multiorgan failure can result.
Although subcutaneous gas is usually present in mixed bacterial infections, it is typically absent in S.
pyogenes and MRSA infections.
Therefore, while imaging techniques, such as X-ray, CT, and MRI may prove useful in visualizing gas
in the tissues, open surgery with tissue evaluation remains the gold standard for diagnosis.
Necrotizing fasciitis may be difficult to differentiate from cellulitis. If it is clinically suspected due to
profound pain, decreased sensation, or skin necrosis, an urgent surgical consultation is essential.
2) Streptococcal pyrogenic exotoxin A.
The most common causative agent for necrotizing fasciitis is Streptococcus pyogenes (also
called group A strep (GAS)). GAS produces a number of toxins that can cause a wide range
of infections. The most commonly involved strains contain M protein types 1, 3, 12, and 28
(usually elaborating pyrogenic exotoxin A) primarily responsible for producing the
symptoms associated with necrotizing fasciitis.
↑
A 7-day-old newborn boy presents to the emergency department with a history of
fever, irritability, and generalized erythema. During the first two days of clinical
manifestations, the child's parents tried to control the symptoms using acetaminophen;
however, the newborn continued to be ill, and blisters were noticeable around the
buttocks, hands, and feet. During the physical examination, his temperature is 39.0°C
(102.3°F), pulse is 130/min, and respirations are 45/min. Ears, nose, and oral mucosa
preserved their integrity, while the skin presents with diffuse blanching erythema and
flaccid blisters with a positive Nikolsky sign.
↑
1) What is the most likely diagnosis in this patient?
2) What are the possible treatment options?
3) What diseases should be considered for differential diagnosis?
1) The newborn in the case presents with clinical manifestations consistent with staphylococcal
scalded skin syndrome (SSSS or Ritter disease), a potentially life-threatening condition caused by
the dissemination of Staphylococcus aureus exfoliative toxins that cause cleavage of desmoglein 1
complex at the zona granulosa at the epidermis, resulting in fragile, tense bullae. It is usually seen
in newborns.
Typical clinical manifestations include fever, irritability, and diffuse blanching erythema that
starts around the mouth before spreading throughout the body. After 2–3 days of the initial
symptoms, fragile blisters appear, gentle pressure to the skin results in separation of the upper
epidermis and wrinkling (Nikolsky sign), and, in severe cases, the entire epidermis may be shed.
2) When SSSS is suspected, cultures from blood, urine, nasopharynx, umbilicus, and other
infectious foci should be obtained.
Treatment requires prompt administration of intravenous antimicrobial therapy with penicillinase-
resistant penicillins (e.g., nafcillin or oxacillin) or vancomycin in cases of resistance.
3) Scarlet fever is one of the nonsuppurative complications of streptococcal tonsillopharyngitis. It
consists of a diffuse erythematous eruption that results from a delayed-type of skin reaction to pyrogenic
exotoxin (erythrogenic toxin, usually types A, B, or C), which is produced by Streptococcus pyogenes.
Pyoderma occurs when S. aureus directly infects the skin of newborns. It is characterized by pustules,
erythematous papules, and honey-colored crust. Lesions usually are found in areas of trauma, such as
the diaper area, axillae, and periumbilical skin. The diagnosis can be confirmed by culturing the fluid
from the lesions.
A 60-year-old woman presents to the emergency
department with pain in her left lower leg. She states that
an insect bit her leg yesterday, and a few hours later she
noticed a sore red spot at the bite location. It steadily
became larger and is extremely painful today. Her past
medical history is significant for insulin-dependent
diabetes and obesity. Her blood pressure is 80/50 mm
Hg, pulse 120/min, respiratory rate 21/min, and
temperature 39.5℃ (103.1℉). Physical exam of the left
lower leg shows erythema, nonpitting edema, and
crepitus with severe pain on palpation of the area. The
serum creatinine is 2.5 mg/dL. Broad-spectrum antibiotic
treatment is initiated. An X-ray of the left lower leg is
shown in picture below.
2) Surgical exploration is appropriate. When crepitus is present or when symptoms are progressing
rapidly, prompt surgical exploration with direct visualization of the fascia and tissues is the diagnostic
standard. A delay in diagnosis can result in amputation or death.
Treatment begins with empirical antibiotic coverage for suspected pathogens and prompt surgical
debridement.
Leg amputation may be required in some patients with necrotizing fasciitis, especially in those with
infections that spread to deeper tissue and involve muscles.
A 7-year-old girl is brought to a clinic with symptoms of fever and sore throat for 2
days. This morning, she developed a rash in her armpits, which is progressing toward
the trunk. The teachers in her school report that none of her classmates have had
similar symptoms. She has a normal birth history. On physical examination, the child
looks healthy. Her pulse is 135/min, respiratory rate 20/min, temperature 39.0°C
(102.2°F), and blood pressure 100/60 mm Hg. An oropharyngeal examination reveals
·
circumoral pallor with a red tongue, as shown in the photograph. The chest and
cardiac examinations are within normal limits. No hepatosplenomegaly is noted.
4) Antibiotics are given primarily to prevent suppurative complications and reduce the likelihood
of ARF, but they do not affect the development of PSGN.
Amoxicillin and penicillin are the treatments of choice. A 1st-generation cephalosporin, such as
cephalexin or cefadroxil, may be substituted for penicillin in cases of penicillin allergy. Alternative
agents are erythromycin and azithromycin (preferred because of better tolerability). Prevention of
ARF depends on eradicating the organism from the pharynx, not simply on the resolution of
symptoms, so 10 days of penicillin treatment are required.
5) Measles is a disease caused by a single-stranded, enveloped RNA virus of the Paramyxoviridae
family. It presents as a prodrome of fever, malaise, and cough, coryza, and conjunctivitis (the 3
Cs), followed by pathognomonic Koplik spots (bright red spots with a blue-white center on the
buccal mucosa) and then by a maculopapular rash. The rash appears approximately 2–4 days after the
onset of fever. The rash spreads from the head to the trunk to the lower extremities. Patients are
considered contagious from 4 days before to 4 days after the appearance of the rash.
Erythema infectiosum is caused by parvovirus B19. The infective period is a few days prior to the
onset of the rash, when patients first have flu-like symptoms. The incubation period for the fifth
disease is 4–14 days. A slapped-cheek appearance and a nonpruritic maculopapular rash over the
extensor surface of the body are the characteristic features of the disease.
Kawasaki disease is a type of vasculitis (inflammation of the blood vessels) affecting medium-sized
arteries throughout the body. The diagnostic criteria for typical Kawasaki disease include:
Conjunctivitis (spares limbus)
Polymorphous rashes on all parts of the body
Cervical adenopathy (> 1.5 cm)
Strawberry tongue
Swollen hands and feet
Number Names for the Etiology Description
disease
1st Measles Measles morbillivirus Cough, coryza, conjunctivitis
disease Rubeola Koplik's spots (blue-white spots with a red halo) on the buccal membrane
Maculopapular rash begins on the face and behind the ears → spreads to trunk/extremities
2nd Scarlet Fever Streptococcus pyogenes Sandpaper-feeling maculopapular rash that begins on the neck and groin → spreads to
disease Scarlatina trunk/extremities
Dark, hyperpigmented areas, especially in skin creases, called Pastia's lines
Strawberry tongue: coated white membrane through which swollen, red papillae protrude
3rd Rubella Rubella virus Asymptomatic in 50% of cases
disease German Fine macular rash on the face (behind the ears) → spreads to the neck, trunk, and extremities
measles (spares palms/soles)
Forscheimer's spots: Pinpoint red macules and petechiae can be seen over the soft palate/uvula
Generalized tender lymphadenopathy
4th Staphylococcal Due to Staphylococcus Begins with a diffuse erythematous rash that usually begins around the mouth → fluid-filled
disease Scalded Skin aureus strains that make bullae or cutaneous blisters → rupture and desquamate
Syndrome epidermolytic Nikolsky’s sign: Applying pressure on the skin with a finger (stroking) results in sloughing off
Filatow-Dukes' (exfoliative) toxin of upper layers.
disease
Ritter's disease
5th Erythema Erythrovirus or parvovirus Facial erythema ("slapped-cheek rash") that consist of red papules on the cheeks
disease infectiosum B19 (Primate Begins on the face → spreads to the extremities → extends to trunk/buttocks
erythroparvovirus 1) Initially confluent, then becomes net-like or reticular as it clears
6th Exanthem Human herpesvirus 6B or Sudden onset of high fever
disease subitum human herpesvirus 7 Nagayama spots: papular spots on the soft palate/uvula
Roseola Rash begins as fever resolves (the term “exanthem subitum” describes “surprise” of rash after
infantum the fever subsides)
Rose rash of Numerous rose-pink, almond-shaped macules on the trunk and neck → sometimes spreads to
infants face/extremities
3-day fever
Skeletal Structure and Function
Week 1 Day 4
A 6-year-old girl has a rash on her face that appeared yesterday. The rash is erythematous and located over
the malar eminences bilaterally. The rash is macular; there are no papules, vesicles, or pustules. A few days
prior to the appearance of the rash, she had a runny nose and anorexia.
Parvovirus B19 is a very small (22 nm) nonenveloped virus with a single-stranded DNA genome. The
genome is negative-strand DNA, but there is no virion polymerase. The capsid has icosahedral symmetry.
There is one serotype.
Laboratory Diagnosis
Fifth disease and aplastic anemia are usually diagnosed by detecting IgM antibodies. B19 virus can be isolated from throat
swabs, but this is not usually done. In immunocompromised patients, antibodies may not be detectable; therefore, viral DNA
in the blood can be assayed by PCR methods. Fetal infection can be determined by PCR analysis of amniotic fluid.
Treatment & Prevention
There is no specific treatment of B19 infection. Pooled immune globulins may have a beneficial effect on chronic B19
infection in patients with immunodeficiencies. There is no vaccine or chemoprophylaxis.
CASE 2
A 24-year-old girl went to a gynaecologist with masses in the vaginal area, the masses were removed and
then referred to a coloproctologist.
PAPILLOMAVIRUSES
Diseases
Human papillomavirus causes papillomas, which are benign tumors of squamous cells (e.g., warts on the
skin). Some HPV types, especially types 16 and 18, cause carcinoma of the cervix, penis, and anus.
Important Properties
Papillomaviruses are nonenveloped viruses with double-stranded circular DNA and an icosahedral
nucleocapsid. Two of the early genes, E6 and E7, are implicated in carcinogenesis. They encode proteins that
inactivate proteins encoded by tumor suppressor genes in human cells (e.g., the p53 gene and the
retinoblastoma [RB] gene, respectively). Inactivation of the p53 and RB proteins is an important step in the
process by which a normal cell becomes a cancer cell.
There are at least 100 types of papillomaviruses, classified primarily on the basis of DNA restriction fragment
analysis. There is a pronounced predilection of certain types to infect certain tissues. For example, skin
warts are caused primarily by HPV-1 through HPV-4, whereas genital warts are usually caused by HPV-6 and
HPV-11. Approximately 30 types of HPV infect the genital tract.
CASE 2,3
PAPILLOMAVIRUSES
Transmission & Epidemiology
Papillomaviruses are transmitted primarily by skin-to-skin contact and by genital contact. Genital warts are
among the most common sexually transmitted diseases. Skin warts are more common in children and
young adults and tend to regress in older adults. HPV transmitted from an infected mother to the neonate
during childbirth causes warts in the mouth and in the respiratory tract, especially on the larynx, of the infant.
Many species of animals are infected with their own types of papillomaviruses, but these viruses are not an
important source of human infection.
Both cell-mediated immunity and antibody are induced by viral infection and are involved in the spontaneous
regression of warts. Immunosuppressed patients (e.g., patients with acquired immunodeficiency syndrome
[AIDS]) have more extensive warts, and women infected with human immunodeficiency virus (HIV) have a
very high rate of carcinoma of the cervix.
CASE 2,3
Clinical Findings
Papillomas of various organs are the predominant finding. These papillomas are caused by specific HPV types.
For example, skin and plantar warts are caused primarily by HPV-1 through HPV-4, whereas genital warts
(condylomata acuminata) are caused primarily by HPV-6 and HPV-11. HPV-6 and HPV-11 also cause
respiratory tract papillomas, especially laryngeal papillomas, in young children.
Laboratory Diagnosis
Infections are usually diagnosed clinically. The presence of koilocytes in the lesions indicates HPV infection. A
polymerase chain reaction (PCR)–based test can be used to detect the presence of the DNA of 14 high-risk
genotypes, including HPV-16 and HPV-18.
Diagnostic tests based on detection of antibodies in a patient’s serum or on isolation of the virus from a patient’s
tissue are not used.
Treatment & Prevention
The usual treatment for genital warts is podophyllin; alpha interferon is also effective and is better at preventing
recurrences than are non-antiviral treatments. Liquid nitrogen is commonly used for skin warts. Plantar warts can
be removed surgically or treated with salicylic acid topically. Cidofovir may be useful in the treatment of severe
HPV infections.
There are two vaccines against HPV. Gardasil, a recombinant vaccine against four types of HPV, contains the
capsid proteins of types 6 and 11, which cause genital warts, and types 16 and 18, which are the two most
common causes of cervical, penile, and anal carcinoma. Cervarix is also a recombinant vaccine but contains the
proteins only of types 16 and 18. Cervarix contains an adjuvant called AS04 that stimulates Toll-like receptors and
thereby enhances antibody production. HPV immunizations have no effect on existing papillomas.
CASE 4
A 10-day-old neonate has several vesicles on the scalp and around the eyes. The child is otherwise well,
afebrile, and feeding normally. A Giemsa-stained smear of material from the base of a vesicle
revealed multinucleated giant cells with intranuclear inclusions.
An 48-year-old man complains of a painful rash on lips. The rash is vesicular and only on that side.
Smear of material from the base of the vesicle reveals multinucleated giant cells with intranuclear
inclusions.
An 80-year-old man complains of a painful rash on his left subcostal region. The rash is vesicular and
only on that side. He is being treated with chemotherapy for leukemia.
The herpesvirus family contains six important human pathogens: herpes simplex virus types 1 and 2, varicella-
zoster virus, cytomegalovirus, Epstein–Barr virus, and human herpesvirus 8 (the cause of Kaposi’s sarcoma).
All herpesviruses are structurally similar. Each has an icosahedral core surrounded by a lipoprotein envelope. The
genome is linear double-stranded DNA. The virion does not contain a polymerase. They are large (120–200 nm in
diameter), second in size only to poxviruses.
Herpesviruses are noted for their ability to cause latent infections. In these infections, the acute disease is followed
by an asymptomatic period during which the virus remains in a quiescent (latent) state. When the patient is exposed
to an inciting agent or immunosuppression occurs, reactivation of virus replication and disease can occur.1 With
some herpesviruses (e.g., herpes simplex virus), the symptoms of the subsequent episodes are similar to those of the
initial one; however, with others (e.g., varicella-zoster virus), they are different.
Three of the herpesviruses, HSV types 1 and 2 and varicella-zoster virus (VZV), cause a vesicular rash, both in
primary infections and in reactivations. Primary infections are usually more severe than reactivations. The other two
herpesviruses, CMV and Epstein–Barr virus (EBV), do not cause a vesicular rash.
CASE 4,5,6
CASE 4,5,6
Four herpesviruses, namely HSV types 1 and 2, VZV, and CMV, induce the formation of multinucleated giant
cells, which can be seen microscopically in the lesions. The importance of giant cells is best illustrated by the
Tzanck smear, which reveals multinucleated giant cells in a smear taken from the painful vesicles of the genitals
caused by HSV type 2
Four herpesviruses, namely HSV types 1 and 2, VZV, and CMV, induce the formation of multinucleated giant
cells, which can be seen microscopically in the lesions. The importance of giant cells is best illustrated by the
Tzanck smear, which reveals multinucleated giant cells in a smear taken from the painful vesicles of the genitals
caused by HSV type 2
HERPES SIMPLEX VIRUSES (HSV)
HSV type 1 (HSV-1) and type 2 (HSV-2) are distinguished by two main criteria: antigenicity and location of
lesions. Lesions caused by HSV-1 are, in general, above the waist, whereas those caused by HSV-2 are below
the waist. Table 37–3 describes some important differences between the diseases caused by HSV-1 and
HSV-2.
TABLE 37–3 Comparison of Diseases Caused by HSV-1 and HSV-2
CASE 4,5,6
Diseases
HSV-1 causes acute gingivostomatitis, recurrent herpes labialis (cold sores), keratoconjunctivitis (keratitis), and
encephalitis, primarily in adults. HSV-2 causes herpes genitalis (genital herpes), neonatal encephalitis and other
forms of neonatal herpes, and aseptic meningitis. Infection by HSV-1 or HSV-2 is a common cause of erythema
multiforme.
Important Properties
HSV-1 and HSV-2 are structurally and morphologically indistinguishable. They can, however, be differentiated by
the restriction endonuclease patterns of their genome DNA and by type-specific monoclonal antisera against
glycoprotein G. Humans are the natural hosts of both HSV-1 and HSV-2.
The typical skin lesion is a vesicle that contains serous fluid filled with virus particles and cell debris. When the
vesicle ruptures, virus is liberated and can be transmitted to other individuals. Multinucleated giant cells are
typically found at the base of herpesvirus lesions.
Immunity is type-specific, but some cross-protection exists. However, immunity is incomplete, and both reinfection
and reactivation occur in the presence of circulating IgG. Cell-mediated immunity is important in limiting
herpesviruses, because its suppression often results in reactivation, spread, and severe disease.
CASE 4,5,6
Clinical Findings
HSV-1 causes several forms of primary and recurrent disease:
• Gingivostomatitis
• Herpes labialis
• Keratoconjunctivitis
• Encephalitis
• Herpetic whitlow is a pustular lesion of the skin of the finger or hand. It can occur in medical personnel as a result
of contact with patient’s lesions.
• Herpes gladiatorum, as the name implies, occurs in wrestlers and others who have close body contact. It is caused
primarily by HSV-1 and is characterized by vesicular lesions on the head, neck, and trunk.
• Eczema herpeticum (Kaposi’s varicelliform eruption) is an infection of the skin of a patient with atopic dermatitis.
Vesicular lesions are seen at the site of the atopic dermatitis (eczema). Most cases occur in children.
• Disseminated infections, such as esophagitis and pneumonia, occur in immunocompromised patients with depressed
T-cell function.
HSV-2 causes several diseases, both primary and recurrent:
• Genital herpes is characterized by painful vesicular lesions of the male and female genitals and anal area
• Neonatal herpes
• Aseptic meningitis caused by HSV-2 is usually a mild, self-limited disease with few sequelae.
CASE 4,5,6
Laboratory Diagnosis
An important diagnostic procedure is isolation of the virus from the lesion by growth in cell culture. The
typical cytopathic effect occurs in 1 to 3 days, after which the virus is identified by fluorescent antibody
staining of the infected cells or by detecting virus-specific glycoproteins in enzyme-linked immunosorbent
assays (ELISAs). HSV-1 can be distinguished from HSV-2 by using monoclonal antibody against glycoprotein
G often in an ELISA test.
A rapid presumptive diagnosis can be made from skin lesions by using the Tzanck smear, in which cells from
the base of the vesicle are stained with Giemsa stain. The presence of multinucleated giant cells suggests
herpesvirus infection.
If herpes encephalitis is suspected, a rapid diagnosis can be made by detecting HSV DNA in the spinal fluid by
using a polymerase chain reaction (PCR) assay. The PCR assay is more sensitive than viral culture. The
diagnosis of neonatal herpes infection typically involves the use of viral cultures or PCR assay.
Serologic tests such as the neutralization test can be used in the diagnosis of primary infections because a
significant rise in antibody titer is readily observed. However, they are of no use in the diagnosis of recurrent
infections because many adults already have circulating antibodies, and recurrences rarely cause a rise in
antibody titer.
CASE 4,5,6
Treatment
Acyclovir (acycloguanosine, Zovirax) is the treatment of choice for encephalitis and systemic disease caused by
HSV-1. It is also useful for the treatment of primary and recurrent genital herpes; it shortens the duration of the
lesions and reduces the extent of shedding of the virus but does not cure the latent state. Acyclovir is also used
to treat neonatal infections caused by HSV-2. Mutants of HSV-1 resistant to acyclovir have been isolated from
patients; foscarnet can be used in these cases.
For HSV-1 eye infections, other nucleoside analogues (e.g., trifluridine [Viroptic]) are used topically. Oral
acyclovir is also used for HSV keratitis. Penciclovir (a derivative of acyclovir) or docosanol (a long-chain
saturated alcohol) can be used to treat recurrences of orolabial HSV-1 infections in immunocompetent adults.
Valacyclovir (Valtrex) and famciclovir (Famvir) are used in the treatment of genital herpes and in the suppression
of recurrences.
CASE 4,5,6
VARICELLA-ZOSTER VIRUS (VZV)
Disease
Varicella (chickenpox) is the primary disease; zoster (shingles) is the recurrent form.
Important Properties
VZV is structurally and morphologically similar to other herpesviruses but is antigenically different. It has a
single serotype. The same virus causes both varicella and zoster. Humans are the natural hosts.
There is infectious VZV in zoster vesicles. This virus can be transmitted, usually by direct contact, to children
and can cause varicella. The appearance of either varicella or zoster in a hospital is a major infection control
problem because the virus can be transmitted to immunocompromised patients and cause life-threatening
disseminated infection.
CASE 4,5,6
Clinical Findings
Varicella
After an incubation period of 14 to 21 days, brief prodromal
symptoms of fever and malaise occur. A papulovesicular rash
then appears in crops on the trunk and spreads to the head and
extremities. The rash evolves from papules to vesicles,
pustules, and, finally, crusts. Itching (pruritus) is a prominent
symptom, especially when vesicles are present. Varicella is
mild in children but more severe in adults. Varicella
pneumonia and encephalitis are the major rare complications,
occurring more often in adults. Reye’s
syndrome, characterized by encephalopathy and liver
degeneration, is associated with VZV and influenza B virus
infection, especially in children given aspirin. Its pathogenesis
is unknown.
CASE 4,5,6
Zoster
The occurrence of painful vesicles along the course of a
sensory nerve of the head or trunk is the usual picture .
The pain can last for weeks, and postzoster neuralgia
(also known as postherpetic neuralgia) can be
debilitating. In immunocompromised patients, life-
threatening disseminated infections such as pneumonia
can occur.
CASE 4,5,6
Laboratory Diagnosis
Although most diagnoses are made clinically, laboratory tests are available. A presumptive diagnosis can be made by
using the Tzanck smear. Multinucleated giant cells are seen in VZV as well as HSV lesions. The definitive diagnosis is
made by isolation of the virus in cell culture and identification with specific antiserum. A rise in antibody titer can be used
to diagnose varicella but is less useful in the diagnosis of zoster.
Treatment
No antiviral therapy is necessary for chickenpox or zoster in immunocompetent children. Immunocompetent adults with
either moderate or severe cases of chickenpox or zoster often are treated with acyclovir because it can reduce the duration
and severity of symptoms. Immunocompromised children and adults with chickenpox, zoster, or disseminated disease
should be treated with acyclovir. Disease caused by acyclovir-resistant strains of VZV can be treated with foscarnet. Two
drugs similar to acyclovir, famciclovir (Famvir) and valacyclovir (Valtrex), can be used in patients with zoster to
accelerate healing of the lesions, but none of these drugs can cure the latent state. There is some evidence that these drugs
reduce the incidence of postzoster neuralgia.
Prevention
There are two vaccines against VZV: one designed to prevent varicella, called Varivax, and the other designed to prevent
zoster, called Zostavax. Both contain live, attenuated VZV, but the zoster vaccine contains 14 times more virus than the
varicella vaccine. The zoster vaccine is effective in preventing the symptoms of zoster, but does not eradicate the latent
state of VZV.
CASE 7
Rubella (German measles) is a member of the Togaviridae family with an approximately 10-kb single-stranded, positive-
sense polyadenylated RNA genome and a lipid envelope.
Humans are the only natural host. Direct person-to-person airborne spread by infected droplets appears to be the usual
mode of transmission. The patient with subclinical infection is also a source of rubella virus. Patients are most contagious
for a few days before and after the onset of rash, although virus may be present in pharyngeal secretions for as long as 1
week before and 2 weeks after the onset of rash.
CASE 7
The clinical manifestations of postnatal rubella range from inapparent infection to a characteristic pattern of adenopathy,
rash, and low-grade fever. The incubation period is from 14 to 21 days. Primary replication occurs in the nasopharynx,
followed by viremia at approximately days 5–7 and rash at days 14–17. A typical clinical course begins with adenopathy
involving primarily the postauricular, occipital, and posterior cervical nodes, which may be slightly painful and tender.
Although symptoms usually clear promptly as the rash fades, nodes may remain palpable for several weeks. Adolescents
and adults may complain of malaise, headache, a low-grade fever, sore throat, and mild coryza during a 1-day to 5-day
prodromal period that frequently accompanies the onset of adenopathy.
The rubella rash is variable but usually brief. It may be no more than a transient blush, but classically it persists for 2 to 3
days in a pattern that has been called kaleidoscopic because of its changing appearance. Initially, small, irregular pink
macules begin on the face and spread rapidly (usually within 24 hours) to the neck, trunk, arms, and ultimately legs. By
the next day, these lesions may have coalesced, developed a maculopapular component, and become scarlatiniform.
An exanthema consisting of punctate or slightly larger red spots on the soft palate may be present during the late
prodrome and early rash phase.
CASE 7
Diagnosis of rubelliform rashes in acutely ill, febrile children and in young adults requires accurate historical information
from parents: vaccine history, source of exposure, prodrome, and progression of rash.
Laboratory testing is required. Rubella can be diagnosed by isolating the virus, detecting viral nucleic acid by polymerase
chain reaction or demonstrating rising titers of rubella antibody in serum. Rubella serum antibody is measured in a variety
of test systems based on neutralization, hemagglutination inhibition, complement fixation, latex agglutination, radial
hemolysis, immunoblot, enzyme immunoassay (EIA), and IgG avidity. Interpretation of antibody testing should always
be guided by clinical, epidemiological, and immunization status data.
Congenital Rubella CASE 7
Epidemiology
Congenital rubella infection is unusual because the readily available vaccine prevents infection in mothers of
childbearing age. However, cases still occur in populations that are not optimally vaccinated. Most cases of congenital
rubella are the result of primary maternal disease during pregnancy. The risk for fetal infection is highest during early
gestation, although the most severe manifestations occur when infection is in the last trimester.
Presentation
Patients born with congenital rubella usually suffer from severe intrauterine growth retardation as well as a variety of
cardiac, ophthalmologic, and neurologic defects. The most common congenital heart defects include patent ductus
arteriosis and pulmonary artery stenosis. Cataracts occur in more than 30% of cases. Severe cases often have “celery
stalk” appearance of long bones visible on plain radiographs. Organomegaly, thrombocytopenia, and purpuric skin
lesions may also be seen. There is considerable clinical overlap in patients with congenital CMV.
Diagnosis
Diagnosis is made by detection of rubella-specific serum IgM antibodies. Congenital infection can also be diagnosed
by documenting increasing serum concentrations of infant rubella IgG over several months. Virus is readily excreted
from throat, urine, and CSF, but viral isolation of rubella is difficult and usually not achieved.
· Rubella IgM
· Persistence of IgG
· Viral culture of throat, urine, CSF
Management
At the present time, there is no treatment for infants with congenital rubella. Defects of the eyes are managed as
needed. Contact isolation is recommended for children with proven or presumed congenital rubella during the first
year of life.
CASE 8
A 6-year-old boy developed rhinorrhea and a mild non-productive cough 8 days before admission to the hospital.
Six days before admission, he developed a tactile fever and was taken to a local clinic. He was prescribed a
mucolytic and an antihistamine. Despite this treatment, his cough worsened and his fever persisted. Three days
before admission, he developed a facial rash, which proceeded to spread to his trunk. On the day before admission
his fever and cough persisted, and he was transferred to Hospital for severe respiratory distress and potential need
for mechanical ventilation. The child had no medical or surgical history. He did not take any medications. His birth
history was unremarkable. He had no known drug allergies. His vaccination history included: BCG vaccine;
diphtheria, pertussis and tetanus vaccine; oral polio virus vaccine (three doses); and hepatitis B vaccine (three
doses).
Complete blood count was notable for haemoglobin (10 grams/dl), haematocrit (29.4%) and a white blood cell
count of 9900 cells/mm3 (neutrophils (41%), eosinophils (4%), lymphocytes (42%), monocytes (3%)). Measles
IgM was reactive. Chest radiograph showed patchy infiltration in bilateral perihilar regions, without pleural
effusion or cardiomegally . Studies were completed 8 days after symptom onset. Blood cultures had no growth at 7
days
Vitamin A oral daily was given for 2 days (200 000 units/day); intravenous amikacin for 3 days (270 milligrams/
day) and cefotaxime for 7 days (2.8 grams/day).
CASE 8
Measles virus is a spherical, nonsegmented, single-stranded, negative-sense RNA virus and a member of
the Morbillivirus genus in the family of Paramyxoviridae.
Measles virus is transmitted primarily by respiratory droplets over short distances and, less commonly, by small
particle aerosols that remain suspended in the air for long periods of time. Direct contact with infected secretions
can transmit measles virus, but the virus does not survive long on fomites.
Infection is initiated when measles virus reaches epithelial cells in the respiratory tract, oropharynx, or conjunctivae.
The incubation period for measles, the time from infection to clinical disease, is approximately 10 days to the onset
of fever and 14 days to the onset of rash. The incubation period may be shorter in infants or following a large
inoculum of virus, and may be longer (up to 3 weeks) in adults.
In most children, the signs and symptoms of measles are highly characteristic. Approximately 10 days after
exposure, fever and malaise signal the onset of illness (Fig. 316-1). Cough, coryza, and conjunctivitis follow
promptly. A gradual worsening of symptoms accompanies a steady rise in fever over the next 4 days. With the onset
of rash 14 days after infection, the clinical picture attains maximal severity. Constitutional symptoms throughout this
10-day period vary, but headache, abdominal pain, vomiting, diarrhea, and myalgia are frequent complaints. Fever
reaching 40°C (104°F) to 41°C (105.8°F), often accompanied by chills, is not unusual when the rash is most florid.
CASE 8
DIAGNOSIS
Measles is readily diagnosed on clinical grounds by clinicians familiar with the disease. Koplik spots are especially
helpful because they appear early and are pathognomonic of measles. The Centers for Disease Control and
Prevention (CDC) case definition for measles requires (1) a generalized maculopapular rash of at least 3 days’
duration; (2) fever of at least 38.3°C (101°F); and (3) cough, coryza, or conjunctivitis.
Serology is the most common method of laboratory diagnosis. A fourfold or greater increase in measles virus–
specific IgG antibody levels between acute and convalescent sera, or the detection of measles virus–specific IgM in
a single specimen of serum or saliva, are considered diagnostic of acute infection
Except for general supportive measures, such as hydration and antipyretics, there is no specific antiviral therapy for
persons with uncomplicated measles. Secondary bacterial infections are a major cause of morbidity and mortality
following measles, and effective case management involves prompt treatment with antibiotics
Skeletal Structure and Function
Week 1 Day 5
A 70-year-old woman presented with chronic ulcerative nodules on her lower extremities at the outpatient clinic.
A tender red nodule developed on her left anterior tibial region a month ago, which gradually enlarged;
spontaneous ulceration of the nodule and occasional pain were reported. The lesions did not respond to topical
antibiotics, and more nodular lesions appeared in the mid-calf area and spread to her toes. Her medical history
included kidney transplantation 6 years ago, chronic tinea pedis and onychomycosis.
Physical examination revealed multiple firm, 2 to 4 cm, verrucose to dome-shaped, red nodular lesions, with an
infiltrative erythematous base on her left lower leg. Some lesions were ulcerated with clear to yellow exudations
and crusts. Several small red nodules were also found on the dorsal side of the toes (Figure a). Biopsies were
obtained from the lesions and microscopic examination revealed pseudoepitheliomatous hyperplasia and
extensive infiltration of neutrophils, plasma cells, histiocytes and multinucleated giant cells in the dermis
(Figure c). PAS stain highlighted fungal hyphae inside a neutrophilic microabscess (Figure d). An examination
of the lesion scales with potassium hydroxide showed hyaline septate hyphae. Fungal culture grew Trichophyton
rubrum.
Treatment
Classification of antifungal cures
Explain pharmocology of antifungal drugs
Skeletal Structure and Function
Week 2 Day 1
↑
extensor surfaces of her upper extremities bilaterally, as shown in the
image. Similar lesions are present on both knees, involving < 8% of
her total body surface area. Laboratory tests are unremarkable.
↑
areas. Pain and itching from the psoriatic lesions can occur, and bleeding may occur if the lesions
are scratched (Auspitz sign).
Psoriasis often has a fluctuating course of remissions and exacerbations. The use of lithium, beta-
blockers, or antimalarials can precipitate a flare-up of psoriasis.
Diagnosis is typically clinical, although a punch biopsy of the skin can confirm the diagnosis if
needed.
2) Common complications include psoriatic arthritis, which occurs in 10–30% of patients with
psoriasis. Psoriatic arthritis is destructive and typically affects the distal joints of the hands and feet.
2) A biopsy of the lesion would show a lymphocytic infiltrate at the dermal-epidermal junction with
the destruction of basal epidermal cells. There is a slight risk of developing squamous cell
carcinoma.
2) EM usually presents 4–10 days after the reactivation of HSV infection and is the
result of IFN-γ release by CD4+ T cells.
An 11-year-old boy was brought in by his mother with red, tender bumps on his
legs. The patient’s mother says that his symptoms started three days ago with a low-
grade fever, malaise, and joint pain. He began to improve over the next two days,
but this morning, she noticed multiple painful red bumps on his shins when he woke
up. Past medical history is significant for a recent severe sore throat and fever one
week ago which resolved without treatment. His temperature is 38.0°C (100.4°F),
blood pressure is 120/70 mm Hg, pulse is 105/min, and respirations are 15/min.
Physical examination of his throat shows slight tonsillar enlargement with mild
erythema but no exudate. Skin exam reveals multiple firm, tender erythematous
nodules with indistinct borders on his anterior shins that are 2–3 cm in diameter.
There is no drainage, bleeding, abscess formation, or ulceration.
2) There are no lab tests specific for EN, but patients may have abnormal test
results due to the underlying disease. Lab tests indicated with EN include a CBC,
ESR, diagnostic test for streptococcal pharyngitis if the patient has signs of acute
pharyngitis, antistreptolysin O titers (repeated 2–4 weeks after infection), and a
chest radiograph in adults to assess for sarcoidosis or tuberculosis. The diagnosis
of EN is usually clinical but can be confirmed with a skin biopsy and
histopathologic analysis.
In pediatric patients with erythema nodosum, a throat or nasal culture for GAS is
indicated, especially in those who have a recent history of fever and sore throat.
Treatment of EN is symptomatic with NSAIDs if needed for comfort. Most EN
resolves spontaneously or with supportive care over days to weeks. The underlying
cause should be treated whenever possible (e.g., antibiotics if a positive culture).
A 4-year-old boy with a rash is brought in by his mother. The patient’s mother says that his
symptoms started acutely a few hours ago after they had eaten shellfish at a restaurant, and the
symptoms have progressively worsened. The rash started with a few bumps on his neck and chest
but quickly spread to involve his arms and upper torso. The patient says the rash makes him
uncomfortable and itches badly. He denies any fever, chills, night sweats, dyspnea, or similar
symptoms in the past. Past medical history is significant for a history of atopic dermatitis at the age
of nine months which was relieved with some topical medications. The patient is afebrile, and his
vital signs are within normal limits. Physical examination reveals the findings seen in the image
below. There is no evidence of laryngeal swelling, and his lungs are clear to auscultation.
Week 2 Day 2
Skin & Subcutaneous Tissue
Adnexal disorders
Learning objectives:
1. Define and discuss etiology and pathophysiology of various adnexal disorders
2. Review the steps involved in the evaluation of various adnexal disorders
3. Discuss the basic principles applied for management of adnexal disorders
MCQ Case 1
A 28-year-old man is referred to the dermatologist after noticing an increasing number of smooth, circular patches of
complete hair loss on his scalp. He says that the involved patches of the scalp have a burning and itching sensation.
There has been no improvement in the condition with the over-the-counter products that were recommended to him by
his hairstylist. He denies pulling his hair intentionally. Physical examination reveals no epidermal inflammation or
erythema, and no fluorescence is detected under Wood’s lamp. A punch biopsy shows a peribulbar lymphocytic
inflammatory infiltrate surrounding anagen follicles, resembling a swarm of bees. Which of the following is the most
likely diagnosis?
A.Tinea capitis
B.Telogen effluvium
C.Lichen planopilaris
D.Androgenic alopecia
E.Alopecia areata
MCQ Case 1
Correct answer E: Alopecia areata, a type of non-scarring hair loss that affects individuals of any age and
sex, with most patients experiencing symptoms before 30 years of age. It is characterized by smooth, circular,
discrete areas of hair loss that are often preceded by a burning sensation or pruritus. The etiology of alopecia
areata is autoimmune, with T cell-mediated peribulbar inflammation that disrupts the normal hair cycle
without scarring (see image below).
Spontaneous regrowth occurs in many patients; however, patients typically present with more than one
episode in their lifetime. Alopecia areata tends to be associated with other autoimmune disorders and
diseases, such as vitiligo, systemic lupus erythematosus, and thyroid disease.
MCQ Case 1
Option A: Tinea capitis is a fungal infection of the scalp by dermatophytes (filamentous fungi of the
genera Trichophyton and Microsporum). It is most commonly seen in children. The most common clinical findings are
pruritus and the presence of scaly patches with alopecia, or patches of alopecia with black dots. Treatment consists of
systemic antifungal medications.
Option B: Telogen effluvium is a form of diffuse, non-scarring hair loss that results from altered regulation of the hair
follicle cycle with a significant amount of hair follicles falling into the telogen phase (resting phase), the end of which
is marked by shedding of hair from the follicle. This results in significant premature shedding and subsequent diffuse,
not localized, hair loss. Causes can be a chronic disease, anemia, pregnancy, and severe emotional distress.
Option C: Lichen planopilaris is a type of scarring alopecia that is characterized by the permanent destruction of hair
follicles. Typically, additional findings such as erythema, scaling, and pustules are present. Some histopathological
characteristics include a band-like lymphocytic infiltrate at the dermal-epidermal junction, vacuolization of the basal
layer, and apoptotic keratinocytes. Treatment with corticosteroids is recommended.
Option D: Hair loss in androgenic alopecia is typically slow, consisting of gradual thinning of hair on the vertex and
frontoparietal areas of the scalp. Up to 50% of patients will develop androgenic alopecia by 50 years of age, and 80%
by 70 years of age. Although histopathological evaluation is rarely required, findings include a mixture of the terminal,
vellus, and vellus-like hair follicles in the dermis.
Learning objective: Alopecia areata is an autoimmune disorder with peribulbar lymphocytic inflammatory infiltrates
surrounding growing follicles that do not produce scarring. Clinically, it is characterized by round, smooth, discrete
areas of hair loss without additional findings (i.e., no erythema, pustules, or scaling).
MCQ Case 2
An 18-year-old man presents to the office complaining of an itchy rash on his torso that appeared one week ago. He is
on the college wrestling team and is concerned that he will not be able to compete if it gets infected. His past medical
history is unremarkable. Vital signs are within normal limits. Physical exam reveals an erythematous, scaly plaque with
central clearing on the left lateral chest wall. Which diagnostic test would be most appropriate at this time?
Correct Answer D: The man's rash is most likely tinea corporis (ringworm), which is caused by dermatophytes
(cutaneous fungi), such as Microsporum and Trichophyton. Potassium hydroxide (KOH) preparation is a fast and easy
way to test for fungal infections in the office. The procedure involves taking a skin scraping from the infected area,
adding KOH solution, and examining the specimen under a microscope. Dermatophytes have branching hyphae, which
confirms the diagnosis of ringworm.
Option A: Sabouraud agar is a growth medium containing peptones used to grow fungi, including dermatophytes, as
well as branching bacteria such as Actinomyces and Nocardia species. Fungi can take a considerable amount of time to
grow in a laboratory. Therefore, the use of this agar is generally limited to serious infections in which fungi are
suspected or the diagnosis is unclear. A cutaneous fungal infection generally does not warrant the use of Sabouraud
agar unless it is refractory to treatment. This man is presenting with an uncomplicated case of tinea corporis, making
use of Sabouraud agar unnecessary.
Option B: Eaton agar is a growth medium that is specific for Mycoplasma pneumoniae. M. pneumoniae is a common
cause of atypical pneumonia, also known as "walking pneumonia" because the symptoms are generally milder, even
though chest X-rays reveal diffuse infiltrates. This man is not presenting with any signs of pneumonia. Additionally, his
cutaneous symptoms cannot be explained by infection with M. pneumoniae, making it an unlikely diagnosis.
MCQ Case 2
Option C: Thayer-Martin agar is a type of growth medium used to isolate Neisseria species. It is a chocolate agar that
contains a combination of antimicrobials known as VCN (vancomycin, colistin, and nystatin) that are used to prevent
the proliferation of other species of bacteria or fungi. This man is not presenting with symptoms indicative of
a Neisseriainfection, which is typically meningitis and sexually transmitted infections. Therefore, Thayer-Martin agar is
of little use in this clinical scenario.
Option E: With Wood’s lamp examination, a healthcare practitioner uses fluorescent light to examine lesions in a
completely dark room. Wood’s lamp exams can help clinicians evaluate hypo- and hyperpigmented lesions, and lesions
of infectious origin. Cutaneous fungal infections often fluoresce a certain color, depending on the cause of infection.
Dermatophytes may glow blue-green or yellow. Wood’s lamp exam findings are very user-dependent and are not used
for the definitive diagnosis of a lesion. Because the test is not sensitive or specific, it is not a confirmatory test and is
inferior to KOH preparation.
Learning objective: Tinea infections are caused by dermatophyte species of fungi, such
as Microsporum and Trichophyton. Fungal skin infections can be diagnosed with KOH preparations or Sabouraud agar,
but KOH preparations are the preferred method to use for uncomplicated infections in the office setting.
MCQ Case 3
Correct answer E: This patient has signs and symptoms strongly suggestive of a fungal infection of the toenail which
is called onychomycosis or tinea unguium. A potassium hydroxide (KOH) preparation of the scraping from his nail
bed will help in the diagnosis of this condition. Diabetes mellitus, psoriasis, and peripheral vascular disease are
common risk factors for fungal infection of nails.
Terbinafine is the drug of choice for the treatment of onychomycosis. It is administered for a prolonged period (> 12
weeks). Other comorbid conditions, which flare up due to treatment with terbinafine, should be considered. Terbinafine
is efficacious when compared to itraconazole, fluconazole, and griseofulvin in the treatment of onychomycosis.
Moreover, terbinafine is fungicidal while others are fungistatic. Adverse effects involve hepatotoxicity/hepatitis.
Continuous monitoring of hepatic transaminases is needed during treatment. It is also an inhibitor of CYP2D6 hepatic
enzymes which results in some drug-drug interactions.
MCQ Case 3
Option A: Amphotericin B is an antifungal agent that has been associated with nephrotoxicity. It is not used until there
is a serious systemic fungal infection. Although terbinafine dose must be adjusted in patients with renal failure, it is not
a nephrotoxic drug.
Option B: Lithium, amiodarone, and Interferon-alpha have been linked to drug-induced hypothyroidism, but not
terbinafine.
Option C: No link has been found between the use of terbinafine and depression. The adverse effects of terbinafine
include rash, dyspepsia, abdominal pain, pruritus, nausea, diarrhea, and taste disturbance.
Option D: Apart from anticancer agents, antibiotics like linezolid and chloramphenicol are known to cause
pancytopenia. Terbinafine does not cause decreased blood cell counts.
Learning objective: Serial monitoring of hepatic transaminases is required in patients receiving terbinafine for
onychomycosis, as it is hepatotoxic.
MCQ Case 4
A 15-year-old boy presents to the clinic complaining of an uncomfortable skin condition that started two years ago.
The patient states that his skin feels "oily" and is embarrassed by his appearance. On examination, he is a healthy-
appearing teenager who has reached the expected Tanner stage. The skin on his face and back has erythema and a few
open and closed comedones. What microbiologic agent is associated with this presentation?
A.HHV-8
B.Streptococcus pyogenes
C.Cutibacterium acnes
D.Human papillomavirus (HPV) strains 6 and 11
E.Bartonella henselae
MCQ Case 4
Correct answer C: Acne vulgaris is a common inflammatory skin disease characterized by comedones and
inflammation. It is caused by increased sebum production and keratinous deposition in the pilosebaceous follicles of the
skin and the involvement of Cutibacterium species, a bacteria normally found on the skin. Acne vulgaris can occur at
any age but most commonly appears during puberty. It can present on any skin area but commonly infects the face and
back. This patient’s age, chronic presentation, and self-description point to infection with C. acnes causing acne
vulgaris.
MCQ Case 4
Option A: Human herpesvirus-8 is a strain of the herpesvirus family that causes Kaposi sarcoma. This disease is an
endothelial malignancy seen in immunocompromised patients; in the United States, it is almost exclusively associated
with human immunodeficiency viruses. Kaposi sarcoma presents as red, purple, or brown raised lesions on the skin and
mouth as well as in the gastrointestinal and respiratory tracts. While the lesions associated with Kaposi sarcoma have a
lymphocytic infiltrate, they are not inflammatory. This patient does not have risk factors for this disease, and his
presentation is not suggestive of this etiologic agent.
Option B: Streptococcus pyogenes causes a variety of soft tissue and skin infections. The infection that presents in a way
that is closest to this patient’s symptoms is erysipelas, which is an acute skin infection. It is a superficial infection of the
upper dermis that affects the skin of the face and the upper and lower extremities. The skin symptoms of raised, sharply
demarcated erythema and mild edema are accompanied by systemic symptoms such as fever, chills, fatigue, headaches,
and vomiting. This is an acute disease process with systemic symptoms that does not match this patient’s presentation.
MCQ Case 4
Option D: Verruca vulgaris, also known as the common wart, is caused by several strains of HPV, especially 6 and 11.
Warts are rough, raised growths that are most often asymptomatic and painless. Histologically, koilocytes and
hyperkeratosis can be seen. While there are many different warts, common warts are most often seen in childhood.
These warts can appear anywhere but are most often seen on the hands. Warts can be chronic but usually resolve by
adulthood. This patient’s symptoms are not suggestive of a diagnosis of verruca vulgaris, making this etiologic agent
unlikely.
Option E: B. henselae is a gram-negative bacteria that causes cat scratch disease. The bacteria enter the skin via a cat’s
scratch (or bite), as the name suggests. There is unilateral lymphadenopathy proximal to the site of trauma followed by
nonspecific systemic symptoms such as fatigue, arthralgias, headache, and chills. Rarely, this can progress to
encephalopathy or meningitis, although most cases resolve. This patient’s presentation is inconsistent with this disease,
making this causative agent unlikely.
Learning objective: Acne vulgaris is a common skin inflammatory condition during puberty caused by the
bacterium Cutibacterium acnes. It is often treated with retinoids and antibiotics.
Skeletal Structure and Function
Week 2 Day 3
1) Define and discuss etiology and pathophysiology of miscellaneous skin and mucosa
disorders, including disorders of pigmentation, traumatic and mechanical disorders,
congenital disorders, adverse effects of drugs on skin and subcutaneous tissue.
2) Review the steps involved in the evaluation of miscellaneous skin and mucosa
disorders.
3) Discuss the basic principles applied for management of miscellaneous skin and
mucosa disorders.
A 32-year-old man of Asian descent presents with a
skin rash after being started on prophylactic doses of
trimethoprim/sulfamethoxazole 3 weeks earlier. He was
diagnosed with acquired immunodeficiency syndrome 2
months ago, prompting the initiation of prophylactic
antibiotics. His blood pressure is 112/72 mm Hg,
temperature is 40.0°C (104.0°F), respirations are
22/min, and pulse is 95/min. He has 8% total body
surface area (TBSA) skin slough with scattered vesicles
and erosions throughout his face and extremities, as
shown in the image. He does have erosions on his lips,
but he has no other mucosal involvement.
Management options:
Airway management
Fluid and electrolyte management
Nutritional support
Pain control
Monitoring and treatment of superinfections
Wound care
A 35-year-old man pulled out of a burning building was found unconscious and
severely injured. He was transported to the nearest emergency department. Upon
arrival, he is stabilized and evaluated for burns and trauma. Approximately 40% of
his body is covered in burns. The burned areas appear blackened and charred. After
awakening, the patient had a loss of pain sensation in the burned areas, and the
lesions did not blanch on palpation. The affected area was soft when palpated.
3) Prognosis and management are based on the patient’s age, the size of the affected area,
evidence of inhalation injury, and the depth of the affected area. Victims of severe burns are
at risk of developing sepsis and acute respiratory distress syndrome due to a profound
inflammatory response. The patient can also have acute renal failure due to severe fluid
loss (burn victims lose the ability to retain water because their skin is damaged). The
metabolic byproducts from substantial muscle breakdown and hemolysis can damage the
kidneys as well.
Degree of burn Characteristics Symptoms Healing
Superficial burn (1st Limited to epidermis Itching to pain Unscarred
degree) No destruction of skin Spontaneous recovery
Hyperemia (red), blanches
with pressure
Edema
Superficial partial burns Limited to epidermis Severe pain Usually unscarred
(2nd degree) Hyperemia Spontaneous recovery
Wet wound bed
Intact sensibility
Blistering
Deep superficial burns Epidermis and dermis Severe pain Partial recovery with scar
(2nd degree) damaged formation
Dry wound bed
Bright and reddened areas
Full-thickness burns (3rd Damage to all skin layers, Painless because nerve Skin regeneration no longer
degree) including superficial fascia endings have been destroyed possible
Grey-white discoloration of Need excision and grafting
skin
No blisters
Full-thickness/eschar Involves muscles, tendons, Painless Skin regeneration no longer
burns (4th degree) or bones possible
Leather-like Need excision and grafting
Charring of tissue
Mild Moderate Severe
All < 2% full thickness 2%–5% full thickness, > 5% full thickness,
high voltage, high voltage,
inhalation, inhalation,
circumferential, circumferential,
comorbid disease comorbid disease
Week 2 Day 4
Musculoskeletal System
Bones, Joints, Tendons, Ligaments and Cartilage
Learning objectives:
1. Discuss the embryologic development of bones, joints, tendons, ligaments and cartilages.
2. Summarize the perinatal changes occurred in bones, joints, tendons, ligaments and cartilage development.
·
·
↑↑
A 2-year-old girl presents to the pediatrician with a history of multiple fractures since birth. The parents report that the
child developed a fractured collarbone after a minor fall at 6 months old and has sustained several other fractures,
including ribs and femurs, with minimal trauma. They also express concern about the child's short stature and delayed
motor development.
Physical examination:
•Vitals: Temperature: 37.0°C (98.6°F), Heart rate: 120 beats/minute, Respiratory rate: 20 breaths/minute, Blood
pressure: 100/60 mmHg
•General: Short stature, blue sclerae, prominent forehead, bowing of long bones
•Musculoskeletal: Limited range of motion in major joints, muscle weakness, joint laxity
1.Identify the most likely diagnosis and outline the differential diagnoses for a child with multiple fractures and blue
sclerae.
2.Briefly explain the underlying genetic defect leading to the presented condition? How does this defect affect the
function of osteoblasts?
3.Identify the major bones affected in presented condition and their anatomical locations.
4.Describe the muscle groups most likely to be affected by muscle weakness in this case and their roles in movement.
5.What would be the group of drugs suggested for presented condition? Discuss the mechanism of action and side
effects.
6.Are there any other pharmacological agents used in the management of presented condition?
↑
•Other Types of Osteogenesis Imperfecta:
• Type I: Usually milder, presenting with fractures later in childhood.
• UPDATE. Type II: Mortal, close to 100% mortality rate. Otherwise death in 1 year after birth.
• Type III: Moderate severity, fractures with minimal trauma but less severe skeletal deformities.
• Type IV: Lethal form with severe bone fragility and deformities, often presenting perinatally.
•Child Abuse: Fractures in atypical locations, inconsistent history, caregiver discrepancies. osteomal
•Nutritional Deficiencies: Vitamin D deficiency (rickets), vitamin C deficiency (scurvy), calcium deficiency. acId
•Metabolic Disorders:
• Homocystinuria: Elevated homocysteine levels can weaken bones.
• Ehlers-Danlos Syndromes: Connective tissue disorders with joint hypermobility and fragile bones.
•Skeletal Dysplasias:
• Achondroplasia: Short stature, disproportionate limbs, kyphosis.
• Osteopetrosis: Increased bone density but paradoxically fragile bones due to abnormal bone structure.
•Genetic Syndromes:
• Marfan syndrome: Tall stature, long limbs, eye abnormalities, aortic valve problems.
High-Yield Points:
•Blue sclerae are highly suggestive of osteogenesis imperfecta but not pathognomonic.
•Consider the severity of fractures, skeletal deformities, and other associated features for further differentiation.
•Carefully evaluate for potential child abuse, especially with discrepancies in history or atypical fracture locations.
•Metabolic disorders and genetic syndromes warrant specific investigations based on clinical suspicion.
•A multidisciplinary approach involving pediatrics, genetics, and orthopedics is crucial for accurate diagnosis and
management.
2. Briefly explain the underlying genetic defect leading to the presented condition? How does this defect affect
the function of osteoblasts?
Genetic Defect:
Osteogenesis imperfecta type III is primarily caused by mutations in the genes encoding type I collagen (COL1A1
and COL1A2). These genes provide the blueprint for building the protein collagen, a crucial component of bone
structure and strength. Most cases are due to autosomal dominant mutations, meaning one altered gene copy from either
parent is sufficient to cause the condition.
Consequences for Osteoblast Function:
The mutated genes lead to the production of defective type I collagen. This collagen is improperly folded, structurally
unstable, and less efficient in forming strong bone matrix. Impaired osteoblast function reduces their ability to
mineralize bone tissue, resulting in decreased bone density and fragility.
Deficient collagen synthesis and abnormal bone matrix disrupt the balance between bone formation and resorption,
further contributing to the bone weakness.
Some mutations may trigger premature osteoblast aging and apoptosis, further diminishing bone production capacity.
3. Identify the major bones affected in presented condition and their anatomical locations.
4. Describe the muscle groups most likely to be affected by muscle weakness in this case and their roles in
movement.
•Long bones and axial structures are often most susceptible due to their high load-bearing roles.
•Muscle weakness typically affects proximal and extensor muscles first due to increased energy demands for movement.
•Weakness in axial muscles can lead to postural problems, decreased respiratory function, and limitations in trunk
movement.
•Weakness in proximal and extensor muscles can impair activities like reaching, grasping, walking, and climbing stairs.
•Early diagnosis and interventions like physiotherapy and muscle strengthening exercises can help prevent or minimize
muscle weakness and improve functional abilities.
5. What would be the group of drugs suggested for presented condition? Discuss the mechanism of action and
side effects.
Bisphosphonates: Bisphosphonates are the mainstay of treatment for increasing bone density and reducing fracture risk.
Mechanism of action: Bind to hydroxyapatite crystals in bone, inhibiting osteoclast activity, reducing bone
resorption, and leading to increased bone density.
Side effects:
• Gastrointestinal upset (nausea, vomiting), heartburn, abdominal pain.
• Acute phase reactions (fever, chills, muscle aches) with intravenous administration.
• Osteonecrosis of the jaw (rare but serious long-term risk).
• Atypical femur fractures (rare but serious, particularly in long-term use).
6. Are there any other pharmacological agents used in the management of presented condition?
Additional Considerations:
Anabolic agents:
• Mechanism of action: Stimulate bone formation by osteoblasts, potentially increasing bone density in some
cases.
• Side effects:
• Acne, mood swings, aggression, liver toxicity.
• Limited use due to safety concerns and potential for virilization in females and precocious puberty in males.
Pain management:
• Mechanism of action: Target pain pathways to alleviate discomfort associated with fractures and
musculoskeletal weakness.
• Side effects:
• Drowsiness, constipation, nausea, dizziness.
• Dependence and addiction risk with extended use of opioids.
A 16-year-old female presents to the emergency department with acute knee pain and swelling after a minor twisting
injury while dancing. She reports a history of frequent ankle sprains, recurrent shoulder dislocations, and generalized
joint hypermobility. Her skin is soft and velvety, stretching easily, and she demonstrates visible hyperextensibility of her
fingers and elbows. Physical examination reveals significant knee effusion and instability, suggesting a potential joint
dislocation.
1. Briefly explain the underlying genetic defect(s) leading to the syndrome and their impact on connective tissue
function, focusing on the most relevant tissues in this case (joints and skin).
·
2. Discuss the rationale and potential effectiveness of non-steroidal anti-inflammatory drugs (NSAIDs) for managing
the patient's acute knee pain and inflammation, considering the possible limitations.
3. Outline the differential diagnoses for the patient's presentation, including other musculoskeletal conditions and
·
potential systemic etiologies that could mimic ligamentous instability and recurrent dislocations.
4. Identify the specific ligaments and capsular structures at the knee joint that are most likely involved in the patient's
suspected dislocation, and explain their anatomical roles in joint stability and movement.
Genetic Defects: Ehlers-Danlos syndrome (EDS) encompasses a group of disorders with diverse genetic mutations
affecting collagen, a crucial protein in connective tissues. This case likely involves either:
• Collagen Synthesis Defects: Mutations in genes encoding specific collagen types (e.g., COL1A1, COL5A1) lead
to abnormal collagen production, compromising its strength and flexibility.
• Collagen Processing Defects: Mutations in genes encoding enzymes involved in collagen processing or cross-
linking disrupt the proper collagen fibril formation, impacting tissue integrity.
•Joint Impact: Deficient collagen affects joint ligaments and capsules, reducing their strength and elasticity. This
predisposes to hypermobility, instability, and recurrent dislocations, as seen in the patient's knee.
•Skin Manifestations: The soft, velvety texture and hyperextensibility of the patient's skin reflect weakened collagen
fibers in the dermis, a characteristic feature of several EDS types.
Differential Diagnoses:
• Hypermobility Spectrum Disorders: Other conditions like hypermobility spectrum disorder (HSD) can mimic
EDS with joint hypermobility but lack the underlying genetic defects and collagen abnormalities.
• Inflammatory Rheumatic Diseases: Inflammatory arthritis like rheumatoid arthritis can also cause joint pain and
swelling, but the absence of systemic inflammatory markers and distinct clinical features help differentiate it
from EDS.
• Congenital Connective Tissue Disorders: Marfan syndrome or Larsen syndrome can share some features like
joint hypermobility, but skeletal and cardiovascular manifestations and distinct genetic profiles will help
distinguish them from EDS.
The knee joint's stability relies on numerous ligaments and the surrounding capsule. Potential structures involved in the
patient's suspected dislocation include:
• Medial Collateral Ligament (MCL): The primary stabilizer against inward knee valgus deformity, commonly
injured in twisting injuries like the one described.
• Anterior Cruciate Ligament (ACL): Controls anterior-posterior stability and can be compromised during sports
injuries or sudden knee hyperextension.
• Posterior Cruciate Ligament (PCL): Prevents hyperextension, which may be more vulnerable in EDS due to its
inherent joint laxity.
• Knee Capsule: The fibrous connective tissue envelope provides overall joint support, and its laxity in EDS
contributes to potential instability.
Skeletal Structure and Function module
Week
2DS
Week 3 Day 2
Musculoskeletal system
A 20-week-old infant is brought to an urgent care clinic by her mother because she has not been eating well for the past 2
days. The mother said her daughter has also been ‘floppy’ since yesterday morning and has been unable to move or open
her eyes since the afternoon of the same day. The child has recently started solid foods, like cereals sweetened with
honey. There is no history of loose, watery stools. On examination, the child is lethargic with lax muscle tone. She does
not have a fever or apparent respiratory distress. What is the most likely mode of transmission of the pathogen
responsible for this patient’s condition?
A.Direct contact
B.Airborne droplet transmission
C.Vector-borne disease
D.Contaminated food
E.Vertical transmission
MCQ Case 1
Correct answer D: The infant most likely has botulism caused by Clostridium botulinum, which is a gram-positive,
anaerobic, spore-forming bacilli most commonly found in soil. Clostridium botulinum can also contaminate
improperly refrigerated food and honey. Clostridium botulinum produces a heat-labile exotoxin called botulinum
toxin that inhibits the release of acetylcholine in the neuromuscular junction, thereby preventing muscle contraction.
Loss of muscle contraction leads to a flaccid type of paralysis.
There are 3 different forms of botulism:
1.Food-borne botulism (mostly from canned foods)
2.Wound botulism (colonization in a wound)
3.Infant botulism (contaminated food, especially honey)
Infants do not have sufficient normal bacterial flora in their gut. When a child ingests the spores of C. botulinum, the
spores colonize the intestinal tract and produce toxins. Infant botulism mainly presents in a previously healthy infant
with symptoms of constipation, weakness in sucking, swallowing, or crying, and muscle weakness (floppy baby). C.
botulinum can be isolated in the stool of an infected infant. Passive immunization with immunoglobulins should be
started as soon as the clinical diagnosis is made. Other therapies include ventilatory support in patients who develop
respiratory paralysis.
MCQ Case 1
Option A: Direct person-to-person contact is a mode of transmission of diseases that occur when microorganisms pass
from an infected host to a healthy person via direct physical contact or through contact with blood or other body fluids.
Examples include touching, kissing, sexual contact, and contact with oral secretions or body lesions. Sexually
transmitted diseases, such as gonorrhea, chlamydia, and syphilis, are examples of infections transmitted via this route.
Option C: Airborne diseases spread via droplets containing the pathogen that are expelled in the air when an infected
person sneezes, coughs, or talks. Measles, influenza, and tuberculosis are a few examples of airborne diseases.
Option D: Vector-borne diseases are human infections caused by bacteria, parasites, and viruses that are transmitted by
mosquitoes, ticks, sandfly mites, and lice. Malaria, dengue, human African trypanosomiasis, schistosomiasis, yellow
fever, and leishmaniasis are some examples of vector-borne diseases. The distribution of vector-borne disease is
determined by demographic, environmental, and social factors.
Option E: Transmission of pathogens from mother-to-baby during, before, or after birth is called vertical transmission
of disease. Transmission can occur through the placenta, breast milk, or through direct contact during or after birth.
Hepatitis B and HIV are common examples acquired through this route.
Learning objective: Infant botulism occurs through the ingestion of honey contaminated with spores of C. botulinum.
As infants do not have sufficient normal flora in their gut, when an infant ingests food containing spores of C.
botulinum, the spores colonize the intestinal tract and produce toxins. The classic presentation is that of a previously
healthy infant with constipation, weakness in sucking, swallowing, crying, and muscle weakness.
MCQ Case 2
A 3-year-old boy presents to the office with his mother because of weakness and difficulty walking. He was born at 39
weeks' gestation via spontaneous vaginal delivery. He is up to date on all vaccinations. He is meeting all verbal and
social milestones, but gross and fine motor skills are delayed. Past medical history is noncontributory. He takes a
multivitamin every day. His mother states that some boys on her side of the family have had similar symptoms and
worries that her son might have the same condition. His blood pressure is 104/62 mm Hg, pulse 90/min, respiratory rate
22/min, and temperature 37.0°C (98.6°F). On physical exam, the boy appears well developed and pleasant. He listens
and follows directions. His heart has a regular rate and rhythm and his lungs are clear to auscultation bilaterally. He
struggles to get up to a standing position after sitting on the floor. Genetic studies reveal a deletion in the gene that codes
for dystrophin. Which of the following is the most likely diagnosis?
Correct answer: A
Duchenne muscular
dystrophy (DMD) is a disease
characterized by muscle
weakness, muscle atrophy, and
dilated cardiomyopathy that
begins before the age of 5 years
and is due to a lack of functional
dystrophin. Dystrophin is a
structural protein of skeletal and
cardiac muscle (see image).
MCQ Case 2
Dystrophin is encoded by the dystrophin gene, which is the largest protein-coding gene in the human genome.
Duchenne muscular dystrophy is an X-linked disorder and follows an X-linked recessive inheritance pattern, so mainly
males are affected. There are thousands of mutations associated with DMD. These mutations either inhibit the
production of dystrophin or significantly modify the protein so that it cannot function.
The muscle weakness in Duchenne muscular dystrophy begins with the muscles of the pelvis and thighs, and the 1st
symptoms are associated with gait. Additionally, patients with this disease often have pseudohypertrophy of the calf
muscles, in which muscle is replaced with fibrous fatty tissue. This boy’s symptoms coupled with his age, sex, and
family history on the mother’s side make Duchenne muscular dystrophy the most likely diagnosis.
MCQ Case 2
Option B: Becker muscular dystrophy is characterized by muscle weakness that begins in adolescence or early
adulthood. Becker muscular dystrophy is similar to Duchenne muscular dystrophy because they both involve mutations
in the dystrophin gene, but the mutation involved in Becker muscular dystrophy is a non-frameshift mutation, often an
insertion, which leads to partially functional dystrophin. Symptoms present later in life and are generally less severe.
This disorder is also X-linked recessive and is seen mostly in males. Although the presentations are very similar, Becker
muscular dystrophy is an unlikely diagnosis because of the age of the boy.
Option C: Limb-girdle muscular dystrophy (LGMD), also known as Erb muscular dystrophy, is a group of disorders
characterized by muscle weakness in both the arms and legs. The muscles of the pelvic girdle and shoulder girdle are
affected first. Usually, there is no calf muscle pseudohypertrophy. As the disease progresses, the skeletal muscles of the
limbs, neck, and back become involved. Eventually, respiratory compromise can occur. The diseases in this group of
muscular dystrophies can be autosomal recessive or autosomal dominant. This boy’s presentation makes this diagnosis
unlikely.
MCQ Case 2
Option D: Myotonic muscular dystrophy is characterized by myotonia (inability to relax muscles), muscle weakness,
and muscle wasting. It is inherited in an autosomal dominant pattern, and symptoms typically present in adulthood.
Additional symptoms include cataracts, balding, and testicular atrophy. The most common form of this disease is
myotonic muscular dystrophy type 1, which primarily affects the distal muscles and is caused by a cytosine–thymine–
guanine (CTG) trinucleotide repeat in the dystrophia myotonica protein kinase (DMPK) gene. This leads to defects in
the expression of myotonin protein kinase. The presentation and age of this boy make this diagnosis very unlikely.
Option E: Emery-Dreifuss muscular dystrophy is a group of muscular dystrophies characterized by muscle
contractures leading to muscle weakness and atrophy. Several genes are involved in the pathogenesis of this disease;
however, the EMD and LMNA genes, which are involved in making the nuclear envelope of the cell, are the most often
involved. There are autosomal dominant, autosomal recessive, and X-linked forms of this disease. Emery-Dreifuss
muscular dystrophy can progress to cardiac involvement that can cause arrhythmia and death. This boy’s presentation
does not involve muscle contracture, making this diagnosis unlikely.
Learning objective: Duchenne muscular dystrophy is a disease characterized by muscle weakness, muscle atrophy,
and dilated cardiomyopathy that begins before the age of 5 years and is due to a lack of functional dystrophin.
Dystrophin is encoded by the dystrophin gene found on the X chromosome. Duchenne muscular dystrophy follows an
X-linked recessive inheritance pattern, so mainly males are affected.
MCQ Case 3
A 45-year-old woman presents to the clinic with weakness that has progressively worsened over the past 2 weeks. She
states that she has a hard time lifting both her arms, but they otherwise function normally. She notes no history of trauma
or other deficits. On examination, she has 2/5 muscle strength on shoulder shrug and arm abduction bilaterally, but the
neurologic exam is otherwise normal. You notice some skin changes and ask her about them. She states that she has had
a rash around her eyes and on her lower face, going down to her neck and chest. The rashes started around the same time
as the weakness began. A complete blood count and basic metabolic panel are normal. Which of the following is the
most likely diagnosis?
A.Myasthenia gravis
B.Polymyalgia rheumatica
C.Lambert-Eaton myasthenic syndrome (LEMS)
D.Dermatomyositis
E.Fibromyalgia
MCQ Case 3
Correct answer: D
Dermatomyositis is an autoimmune disease characterized by symmetric progressive proximal muscle weakness. It is
caused by inflammation of the perimysium of the muscle, a connective tissue sheath surrounding a bundle of muscle
fibers, by CD4+ T cells. Serum markers include an increased creatine kinase level, as well as a positive antinuclear
antibody, positive anti-Jo-1, and positive anti-SRP (signal recognition particle) antibodies. Dermatomyositis can be
inherited or can be triggered by a viral infection; it can also be associated with underlying cancer, most commonly
ovarian, breast, or lung.
MCQ Case 3
Dermatomyositis and polymyositis are
similar in presentations of muscle
weakness; however, there are significant
differences. Polymyositis causes
inflammation of the endomysium of the
muscle, which is a layer of areolar
connective tissue that ensheathes each
individual myocyte, by CD8+ T cells. It
also does not produce cutaneous
symptoms. Dermatomyositis presents
with cutaneous symptoms such as malar
rash, Gottron papules, periorbital
(heliotrope) rash, and a "shawl rash" that
covers the neck and chest (see images
below).
This woman’s symptoms of proximal
muscle weakness and cutaneous
symptoms make dermatomyositis the
most likely diagnosis.
MCQ Case 3
MCQ Case 3
MCQ Case 3
Option A: Myasthenia gravis is an autoimmune disease caused by autoantibodies to the postsynaptic nicotinic
acetylcholine (ACh) receptors at the neuromuscular junction (NMJ). When the ACh receptors are blocked or destroyed
by autoantibodies, the nerve impulses cannot be propagated, leading to muscle weakness. Because this occurs at the
NMJ, skeletal muscle is most affected. Clinical presentation includes fatigue, muscle weakness that worsens with use,
ptosis, and diplopia. Acetylcholinesterase inhibitors can improve symptoms because they increase the amount of
available ACh. Myasthenia gravis symptoms do not include cutaneous symptoms, making this diagnosis unlikely.
Option B: Polymyalgia rheumatica is a syndrome characterized by joint pain and muscle stiffness with systemic
symptoms such as fever and fatigue. It is highly associated with giant cell arteritis, an autoimmune-mediated vasculitis
that typically affects the temporal artery. There is no muscle weakness or rash associated with this condition. Both
polymyalgia rheumatica and giant cell arteritis present more commonly in older women. Because polymyalgia
rheumatica does not affect muscle strength and does not cause a rash, it is an unlikely diagnosis in this woman.
MCQ Case 3
Option C: LEMS is an autoimmune disease characterized by autoantibodies to presynaptic calcium channels in the
neuromuscular junction. Calcium cascade is responsible for the release of acetylcholine from presynaptic vesicles;
therefore, this disease causes a decrease in ACh availability, resulting in muscle weakness. Clinical presentation
includes proximal muscle weakness that improves with muscle use, as well as sparing of the eyes (no diplopia or
ptosis). Weakness is more prominent in the lower extremities than in the upper extremities. LEMS most often occurs as
a paraneoplastic syndrome associated with small cell lung cancer. This woman’s presentation does not suggest LEMS or
small cell lung cancer, making this diagnosis unlikely.
Option E: Fibromyalgia is a disorder characterized by chronic and diffuse joint and muscle pain. Other associated
symptoms include paresthesias, tender points, fatigue, and sleep disturbances. The underlying cause is unknown. The
disorder is more common in middle-aged women, suggesting a possible autoimmune etiology. Fibromyalgia is generally
a diagnosis of exclusion, but also includes a number of characteristic tender points. This woman is exhibiting specific
and symmetric weakness, suggesting that fibromyalgia is not a likely diagnosis.
Learning objective: Muscle weakness has many causes, including anemia, electrolyte imbalance, trauma, autoimmune
disorders, and genetic disorders. Looking at a patient’s history and conducting a physical exam helps narrow the
possible causes of weakness.
MCQ Case 4
The patient presented with intermittent abdominal pain. A preoperative diagnosis of juxta-visceral aortic
aneurysm was made, and he was treated with an axillo-uni-iliac and visceral bypass with spliced Dacron 8
mm to 8 mm polytetrafluoroethylene (PTFE) bifurcated graft with subsequent explantation. On
postoperative day 7, abdominal distension was noted.
Which of the following is the most likely diagnosis?
A.Intraabdominal bleeding
B.Gastrointestinal bleeding
C.Postoperative ileus
D.Mesenteric cyst
E.Abdominal hernia
Correct answer: C
Postoperative ileus (POI) is defined the reduction of gastrointestinal (GI) motility after abdominal or other
types of surgery and is characterized by abdominal distension, lack of bowel sounds, accumulation of gas and
fluids in the bowel, and delayed passage of flatus and stools. Each segment of the gastrointestinal tract
recovers activity at a different rate after surgical manipulation. The small intestine recovers motility within
several hours, the stomach within 24 to 48 hours, and the colon in 3 to 5 days. However, in postoperative
period, some patients experience a prolonged inhibition of coordinated bowel activity that causes
accumulation of secretions and gas, resulting in nausea, vomiting, abdominal distension, and pain.
While NSAIDs can effectively manage pain and inflammation in acute knee injury, their use in EDS requires caution
due to several factors:
• Increased Bleeding Risk: EDS patients may have platelet abnormalities leading to increased bleeding risk,
further exacerbated by NSAIDs that inhibit platelet aggregation.
• Gastrointestinal Complications: NSAIDs can irritate the gastrointestinal tract, especially in EDS patients due to
potentially weaker connective tissues in the stomach and intestines.
• Limited Long-Term Benefits: NSAIDs provide symptomatic relief but don't address the underlying joint
instability in EDS. Alternative approaches like physical therapy and bracing may be more beneficial in the long
term.
Skeletal Structure and Function module
Week 3 Day21 4
Musculoskeletal system
B.HLA-DR2
C.HLA-DR5
D.HLA-DQ2
E.HLA-B27
MCQ Case 1
Correct answer A: This patient most likely has rheumatoid arthritis (RA). She is presenting with a series of classic
symptoms, including symmetric arthralgia, morning stiffness, fatigue, and a moderately positive Anti-cyclic citrullinated
peptide antibody (anti-CCP antibody) or ACPA. ACPA is more specific than rheumatoid factor (RF), and patients can be
RF negative and yet have RA if they meet the following American College of Rheumatology criteria:
Joint involvement 1 large joint 0
2 to 10 large joints 1
1 to 3 small joints (with or without the involvement of large joints) 2
Option B: HLA-DR2 is associated with multiple sclerosis, hay fever, SLE, and Goodpasture's syndrome, not RA.
Option C: HLA-DR5 is associated mainly with pernicious anemia. Findings include anti-parietal cell and anti-intrinsic
factor antibodies, which lead to vitamin B12 deficiency and, subsequently, megaloblastic anemia.
Option D: HLA-DQ2 is associated with celiac disease. The body reacts to the protein gliadin found in most bread and
other products containing wheat. Celiac disease is associated with such positive findings as elevated IgA anti-tissue
transglutaminase and anti-endomysial, anti-deamidated gliadin peptide antibodies titers. Treatment consists of a gluten-
free diet.
Option E: Several types of arthritis are associated with HLA-B27. These are referred to as seronegative
spondyloarthropathies. The most significant of these is ankylosing spondylitis, which is seen in adult men more than
women. It classically involves the sacroiliac joints and leads to ankylosis of the spine characterized on X-ray as
"bamboo spine". Treatment is mainly physiotherapy and pain management. Specific treatment can consist of therapeutic
antibodies such as adalimumab, infliximab, and etanercept. Reactive arthritis, enteropathic arthritis, psoriatic arthritis,
Behçet disease, and juvenile idiopathic arthritis are other types of seronegative arthropathies.
Learning objective: Major histocompatibility complex defects are associated with a series of conditions. Rheumatoid
arthritis, in particular, is a type III hypersensitivity reaction associated with the HLA-DR4 subtype.
MCQ Case 2
A 35-year-old woman presents for evaluation of symmetric proximal muscle weakness. She has a blue-purple
discoloration of the upper eyelids and also a pruritic, papular rash on the knuckles, as shown in the photo. What is the
most likely diagnosis?
A.Polymyositis
B.Duchenne muscular dystrophy
C.Hypothyroidism
D.Inclusion body myositis
E.Dermatomyositis
MCQ Case 2
Option A: Polymyositis is an inflammatory myopathy. A history of proximal muscle weakness and physical
examination with no upper motor neuron signs bring this to the differential diagnosis. General lab testing plus creatine
kinase and serologic testing for myositis-specific autoantibodies clarify the diagnosis. If still unclear, electromyography
and nerve conditions studies are helpful. Muscle biopsy histology shows muscle fiber necrosis, degeneration,
regeneration, and inflammatory cell infiltrationa.
Option B: Duchenne muscular dystrophy is not associated with Gottron’s papules.
Option C: Hypothyroidism is not associated with Gottron’s papules.
Option D: Inclusion body myositis occurs in patients > 50 years of age. Affected patients present with weakness of the
distal muscles rather than proximal muscles.
Learning objective: Dermatomyositis is an inflammatory myopathy associated with Gottron’s papules and a heliotrope
rash on the eyelids.
MCQ Case 3
A 13-year-old girl is admitted to the hospital due to muscle weakness, pain, and arthralgia in her wrist joints. The patient
says, "I am having trouble walking home after school, especially climbing steep hills." She also complains of malaise.
On physical examination, a heliotrope rash is observed around her eyes, and multiple hyperkeratotic, flat, red papules
with central atrophy are present on the back of the metacarpophalangeal and interphalangeal joints. Deposits of calcium
are also noted on the pads of her fingers. Her serum creatine kinase levels are elevated. Which of the following
antibodies is most likely to be found in this patient?
A.Anti-Sm
B.Anti-Jo-1
C.Anti-centromere
D.Anti-Scl-70
E.Anti-histone
MCQ Case 3
Correct answer B: This patient has classic findings of dermatomyositis, an idiopathic inflammatory myopathy with
very specific cutaneous findings (i.e., heliotrope erythematosus periorbital rash and Gottron papules on the hands).
Laboratory findings may include increased creatine kinase due to muscle damage, and antinuclear antibodies, including
anti-Jo-1, anti-SRP, and anti-Mi-2.
Treatment consists of immunosuppressant therapy with methotrexate or corticosteroids. Mild cutaneous symptoms can be
treated with topical corticosteroids and by avoiding ultraviolet light exposure. Other autoimmune diseases also present
with specific antibodies, the findings of which are key for diagnosis in combination with clinical symptoms. Antibodies
found in autoimmune diseases include the following:
•Systemic lupus erythematosus (SLE) — antinuclear, rheumatoid factor, anti-dsDNA, antiphospholipid, and anti-Sm
•Drug-induced lupus erythematosus — anti-histone
•Rheumatoid arthritis — anti-citrullinated peptide and rheumatoid factor
•Polymyositis and dermatomyositis — antinuclear, anti-Jo-1, anti-SRP, and anti-Mi-2
•Systemic sclerosis — anti-centromere, anti-Scl-70, and rheumatoid factor
•Sjögren’s syndrome — anti-Ro and anti-La
•Overlap syndrome or mixed connective tissue disorder — anti-U1 RNP
MCQ Case 3
The Myositis Association Criteria advise that patients should present with at least 4 musculoskeletal symptoms/signs
and at least 1 skin symptom from the lists below. Additionally, a polymyositis diagnosis can be established if the patient
has no skin lesions.
The diagnostic criteria for dermatomyositis are as follows.
Musculoskeletal symptoms/signs (at least 4):
•Proximal muscle weakness — upper or lower extremity and trunk
•Elevated serum CK or aldolase level
•Muscle pain (grasping or spontaneous pain)
•Myogenic changes on electromyography — of short duration and polyphasic motor unit potentials with spontaneous
fibrillation potentials
•Positive anti-Jo-1 (histidyl-tRNA synthetase) antibody
•Non-destructive arthritis or arthralgias
•Systemic inflammatory signs — fever > 37.0°C (98.6°F) at the axilla, and elevated serum CRP level or accelerated
ESR (erythrocyte sedimentation rate) of > 20 mm/h (Westergren method)
•Pathological findings compatible with inflammatory myositis — e.g., inflammatory infiltration of skeletal evidence of
active regeneration
Skin lesions (at least 1):
•Heliotrope rash — red-purple edematous erythema on the upper palpebra
•Gottron’s sign — red-purple keratotic, atrophic erythema, or macules on the extensor surface of finger joints
•Erythema on the extensor surface of extremity joints — slightly raised red-purple erythema over elbows or knees
MCQ Case 3
Learning objective: Dermatomyositis is an idiopathic inflammatory myopathy with cutaneous findings such as
periorbital rash and Gottron’s sign. Laboratory findings may include increased creatine kinase levels due to muscle
damage, and antinuclear antibodies, including anti-Jo-1, anti-SRP, and anti-Mi-2. Dermatomyositis can be managed
with corticosteroids.
MCQ Case 4
A 32-year-old woman presents to her primary care provider complaining of a psoriatic flare that has worsened over the
past 2 days. The patient states that her psoriasis is normally well controlled. She also complains of some fatigue and
states that she has recently developed pain and tenderness in the joints of her hands, with the right hand being more
tender than the left. Her hands are stiff in the morning, and sometimes her fingers swell up. She attributes these changes
to her new job, where she constantly uses her hands to manufacture cabinets. Physical examination reveals plaques with
silvery scale on her elbows and knees. The distal joints of her right hand are mildly swollen, and the nails on both hands
appear pitted. What is the most likely pathogenesis of her joint pain?
Correct answer E: This patient is presenting with some signs and symptoms of psoriatic arthritis, a seronegative
spondyloarthropathy characterized by multiple joint arthritis in patients with psoriasis. Psoriatic arthritis is seen in a
small percentage of patients with psoriasis. It is an autoimmune inflammatory condition of the joints that is negative for
serum markers of rheumatoid arthritis. Psoriatic arthritis is believed to result from the release of TNF and activation of
the RANKL pathway. Activation of the RANKL pathway leads to focal erosions in the bone through stimulation of
osteoclasts.
MCQ Case 4
Option A: Septic arthritis is an infection of joint spaces that causes redness, swelling, decreased range of motion, and
pain in the infected joint. It is an acute infectious process with a progression of symptoms in the joint and does not
remit. The large joints, such as the knees, are most commonly involved. Staphylococcus aureus is a common cause. This
patient is at increased risk for a soft tissue infection due to chronic skin disease. However, her clinical presentation is
not suggestive of infection.
Option B: Osteoarthritis is a common degenerative disease of the joints, most commonly affecting the knees, lumbar
vertebra, and distal interphalangeal and proximal interphalangeal joints of the hands. Osteoarthritis is most often
associated with wear-and-tear injuries that are related to repetitive movements and aging, but it is also associated with
obesity and trauma. Symptoms often include aching pain that is worse with use and with certain motions. Diagnosis is
usually clinical but can be confirmed by x-ray, which shows a narrowing of the joint spaces and degeneration of the
articular cartilage. This patient’s presentation is more consistent with psoriatic arthritis.
MCQ Case 4
Option C: Rheumatoid arthritis (RA) is a systemic autoimmune disease most commonly seen in middle-aged women.
Joint destruction occurs due to the degradation of the articular cartilage and the fusion of the affected joints. Early in the
disease, it can be difficult to delineate RA from osteoarthritis. RA is accompanied by a variety of nonspecific symptoms
such as fever, fatigue, and weight loss. This patient’s presentation is more consistent with psoriatic arthritis.
Option D: Gout is a condition in which excess uric acid in the blood is deposited in body tissues and joint spaces as
negatively birefringent monosodium urate crystals. Uric acid is a byproduct of purine metabolism. Joint aspiration with
visualization of these crystals is the only definitive diagnosis. Gout can be both acute and chronic. Acute gout attacks
will present with severe pain to a particular joint, most commonly the great toe (podagra). Gout is unlikely in this
patient.
Learning objective: Psoriatic arthritis is a seronegative spondyloarthropathy associated with the skin disease psoriasis.
It is an autoimmune inflammatory arthritis, and it is believed to result from the release of TNF and activation of the
RANKL pathway. Activation of the RANKL pathway leads to focal erosions in the bone through stimulation of
osteoclasts.
MCQ Case 5
A 62-year-old carpenter presents to your clinic complaining of worsening joint pain in her hands. She states that the pain
is present in all of her fingers, but is worse in the right hand and gets better when she has a few days off from work. She
denies any paresthesias, fevers, or fatigue. Vital signs are normal. Physical exam reveals hard, non-tender lesions
overlying the proximal interphalangeal joints of the 2nd and 3rd fingers of the right hand. All five digits of the right
hand have a decreased range of motion. An X-ray of her hands is shown. What is the most likely pathogenesis
underlying the X-ray findings?
Correct Answer A: Chronic degradation of articular cartilage. The woman has osteoarthritis, which is a common
degenerative disease of the joints that most commonly affects the knees, lumbar vertebra, and distal (DIP) and proximal
interphalangeal (PIP) joints of the hands. It is most often associated with wear-and-tear injuries that are related to
repetitive movements and aging, as well as with obesity and trauma.
Symptoms often include aching joint pain that worsens with use as well as a limited range of motion. The nodules
described in this woman’s history are Bouchard’s nodes (of the PIP joints, and Heberden's nodes appear on the DIP
joints), which are only found in osteoarthritis and not rheumatoid arthritis. Diagnosis is usually clinical but can be
·
confirmed with X-rays, which would show a narrowing of the joint spaces due to the degeneration of the articular
cartilage in the joints.
-
MCQ Case 5
Option B: Overgrowth of gram-positive bacteria indicates septic arthritis, which is an infection of the joint spaces that
causes redness, swelling, a decreased range of motion, and pain in the joint. It is a monoarticular (single joint) disease,
and large joints, such as the knees, are the most commonly involved. Staphylococcus aureus, a gram-positive coccus, is
the most common bacterial cause of septic arthritis. Bacterial arthritis can be the result of seeding from a superficial
infection, from trauma due to a penetrating wound, or from bacteremia. It is an acute infectious process with a
progression of symptoms in the joint that does not remit.
Option C: Deposition of monosodium urate (uric acid) crystals occurs with gout, which is sudden in onset and typically
monoarticular. Uric acid is produced by the metabolism of purine; when there is an excessive amount of uric acid in the
blood, it can precipitate in synovial fluid as crystals and cause severe pain and swelling of the joint. Definitive diagnosis
is by synovial fluid analysis, and the uric acid crystals are needle-shaped and negatively birefringent when exposed to
polarized light. Gout can be acute or chronic. Acute gout attacks present with significant pain involving a particular
joint, most commonly the big toe (podagra). These attacks are often triggered by the ingestion of alcohol or meat.
Chronic gout results in the formation of tophi and renal failure.
MCQ Case 5
Option D: Genetic predisposition associated with HLA-B27 highly correlates with the seronegative
spondyloarthropathies, which are characterized by inflammatory arthritis of the axial skeleton (sacroiliac joints and
vertebral column) and enthesitis (inflammation of tendon and ligament insertions). They include ankylosing spondylitis,
reactive arthritis, psoriatic arthritis, and inflammatory bowel disease-associated arthritis. Enthesitis sets the negative
spondyloarthropathies apart from other forms of arthritis and usually involves the calcaneal tendon. Involvement of
other systems, such as ocular inflammation (uveitis), may also be involved.
Option E: Production of rheumatoid factor and other autoantibodies is seen with rheumatoid arthritis (RA), a systemic
autoimmune disease that most commonly occurs in middle-aged women. Autoantibodies to the Fc portion of IgG
(rheumatoid factor) are the characteristic laboratory findings in RA. It can be difficult to differentiate RA from
osteoarthritis early in the disease, but RA typically presents as pain and joint stiffness in the morning that improves with
movement, unlike osteoarthritis, and the PIP joints are affected but the DIP joints are spared. RA is also accompanied by
a variety of non-specific symptoms such as fever, fatigue, and weight loss. As the disease progresses, various organ
systems are affected, leading to vasculitis, pleural effusion, pericarditis, and interstitial lung fibrosis.
Learning objective: Osteoarthritis is the most common cause of arthritis. It is a chronic joint condition that is relieved
with rest. X-ray findings are consistent with joint space narrowing due to articular cartilage degradation. The causes of
osteoarthritis include wear and tear, aging, obesity, and previous trauma.
MCQ Case 6
A 33-year-old man presents to the clinic complaining of multiple painful joints for the past 2 weeks. The patient notes
no history of trauma or any joint disorders. The patient states that he is generally healthy except for a recent emergency
room visit for severe bloody diarrhea, which has resolved. On further questioning, the patient admits to some discomfort
with urination but notes no recent sexual activity. On examination, the patient is not in acute distress, with no joint
deformity, evidence of trauma, swelling, or erythema. He has a decreased range of motion of his right knee secondary to
pain. Vital signs are as follows: heart rate 75/min, blood pressure 120/78 mm Hg, respiratory rate 16/min, and
temperature 37.3°C (99.0°F). What is the next step in the treatment of this patient?
Correct answer B: This patient is presenting with reactive arthritis (formerly known as Reiter’s syndrome), a
seronegative spondyloarthropathy. This disease is characterized by inflammatory polyarthritis that results after a
gastrointestinal infection (e.g., Shigella, Salmonella, Yersinia enterocolitica, or Campylobacter jejuni) or chlamydia
infection.
The initial infection results in an autoimmune response that causes arthritis, uveitis, and urethritis. Reactive arthritis is
associated with HLA-B27. Treatment of reactive arthritis involves the treatment of any underlying active infection with
NSAIDs or immunosuppressants to address inflammatory arthritis. Because this patient likely had a gastrointestinal
infection due to his history, no treatment is necessary, since his diarrhea has resolved. Treatment for this patient would,
therefore, center around supportive care and decreasing inflammation.
MCQ Case 6
Option A: Intravenous antibiotics are indicated in patients with serious infections. In this case, the most likely
infections would be septic arthritis or osteomyelitis. Septic arthritis is the infection of a joint space. It can begin from a
local infection, puncture wound, or bacteremia. This patient’s presentation includes fever as well as erythema, warmth,
and swelling in the affected joint. Osteomyelitis is an infection of the bone itself, most often caused by Staphylococcus
aureus. The presentation also includes fever, localized pain, erythema, and swelling. Both of these infections can cause
the patient to become acutely ill. This patient is not presenting with symptoms consistent with an acute, localized
infection, making treatment with IV antibiotics a poor choice.
Option C: PET scanning is a nuclear medicine imaging technique that is used to visualize metabolic processes in the
body. PET scans can identify areas of the body with increased fludeoxyglucose uptake, and therefore is used primarily
to identify malignancies in the body. PET scans can also be used to reveal active infections by identifying the body’s
inflammatory response to the infection. This patient does not have a currently active infection or a high likelihood of
malignancy. A PET scan is therefore not indicated.
MCQ Case 6
Option D: Rheumatoid factor is an autoantibody to the Fc portion of IgG and is the characteristic lab finding for
rheumatoid arthritis. Rheumatoid arthritis (RA) is an autoimmune disease with joint destruction due to the degradation
of the articular cartilage and fusion of the affected joints. Early in the disease, it can be difficult to delineate RA from
osteoarthritis. RA is accompanied by a variety of non-specific symptoms such as fever, fatigue, and weight loss are
often present. As the disease progresses, various organ systems are affected, leading to vasculitis, pleural effusion,
pericarditis, and interstitial lung fibrosis. The disease is seen most often in middle-aged women. This patient has no
signs or symptoms of RA, making testing for rheumatoid factor an unlikely way to aid in the diagnosis of his disease.
Option E: A prostate biopsy is indicated in patients who have a high likelihood of prostate cancer or symptoms of
chronic prostatitis. Prostate cancer is commonly seen in men over the age of 65 years and is associated with metastasis
to the lower back, causing chronic back pain. Discomfort with urination is not a common symptom of prostate cancer
but can be associated with chronic prostatitis. Chronic prostatitis is associated with urinary symptoms such as dysuria,
urgency, and frequency, as well as suprapubic or abdominal pain and pain with ejaculation. Prostatitis is not associated
with joint pain. This patient is at very low risk for prostate cancer and therefore a prostate biopsy is not indicated.
Musculoskeletal System
Neoplasms of the Bone
Learning objectives:
1. Define and discuss common benign and malignant bone tumors.
2. Discuss the secondary bone tumors and pathophysiology of them.
MCQ Case 1
A 58-year-old, previously healthy Caucasian man visited our emergency
department after stumbling in a local pub. He complained of pain in his left
upper leg and was not able to bear weight on it.
An examination showed a large swelling on his upper leg that was very tender
upon palpation, and his leg was shortened.
Radiographs showed a comminuted spiral
fracture of the femoral shaft
Conclusion?
Initially, the femoral shaft fracture had a pathological nature.
Several facts speak for this.
1. The patient stumbled in a local pub. Such a traumatic
mechanism is unlikely to cause the femoral shaft fracture
presented;
2. A primary tumor existed at the site of the fracture and was
spread distally by the insertion of the intramedullary nail.
3. The initial trauma imaging studies did not reveal any signs
of a pathological fracture, but the last radiographs showed
lytic lesions in different fracture fragments.
According to these, the osteosarcoma in the distal femur would
be an iatrogenic skip metastasis.
MCQ Case 2
A six-year-old female patient presented with pain in the right hip of four
months duration, along with difficulty in walking.
There was no other significant contributing history.
On local examination, there was tenderness on deep palpation of the right hip
and restricted range of hip movements. The overlying skin was normal with no
redness or dilated veins.
Plain radiographs revealed a well-defined, expansile,
lytic lesion involving the proximal portion of the right
femur in the trochanteric and subtrochanteric region
approximately 5 cm x 5 cm in size
An MRI of both hips was done,
which showed a hyperintense
lesion in the proximal end of the
shaft of the right femur with
internal septations on a T2-
weighted image. It appeared
hypointense on the T1-weighted
image and showed
inhomogenous enhancement on
the contrast study, which was
suggestive of an aneurysmal
bone cyst
The patient was prepared for surgery. We aspirated the lesion
under the guidance of an
image-intensifier and then injected
polidocanol, a sclerosing agent,
percutaneously. This was augmented
by fixation with two ender’s nails
for prophylactic stabilization of the
affected region. It was sent for
histopathological examination,
which confirmed it to be an
aneurysmal bone cyst.
Based on the history, physical exam, and imaging results, the patient was admitted
and a PET scan was performed.
It revealed a large soft tissue mass occupying the right hemithorax and invading the
anterior chest wall and the overlying ribs, which is consistent with Ewing sarcoma.
No evidence of FDG-avid disease in the rest of the body was appreciated.
Thereafter, the decision was made to perform a CT-guided biopsy. The
procedure was carried out under sedation, and 5 fragments of soft tissue
measuring in aggregate 0.7 × 0.5 × 0.2 cm were submitted to the pathology
lab.
Histopathology revealed a small-blue-round cell tumor. The pathology result
was consistent with Ewing sarcoma.
Treatment:
Chemotherapy
The surgery involved the resection of part of the 5th rib, total resection of the
6th rib, right lower lobe wedge resection, and partial resection of the right
middle lobe. In addition, two other pulmonary nodules were detected in the
right middle lobe and were removed. Segmental lung resection had to be
performed on the stuck right lower lobe, with pleurectomy as an en bloc with
the whole specimen
This is the en bloc resected
specimen showing the
involved ribs completely
resected (on the left side) with
the residual Ewing sarcoma
tumorous mass, including the
thoracic muscles (on the right
side of the picture), resected
to secure free margins.
MCQ Case 4
A postmenopausal married woman was admitted to the department of
gynecology at the age of 61 due to abnormal uterine bleeding. There was
not any specific past medical and familial history and the patient did not
take any specific medications. She mentioned that the bleeding was
presented as occasional spotting. The physical examinations including the
physical examination of the vagina, cervix, and uterus, were all normal. A
pipelle biopsy reported the presence of adenocarcinoma, and therefore,
the patient underwent total abdominal hysterectomy and bilateral
salpingo-oophorectomy (TAH-BSO), subsequently. The pathologic report
showed grade I adenocarcinoma that cancer cells had spread halfway or
more into the myometrium without any extension outside of the uterine
without extension outside the body of the uterus. The tumor was located
at the fundus of it (FIGO stage IB). The patient underwent observation
alone.
Six months after surgery, she was referred to the department due to
new-onset bone pain at her pelvic region. Physical examination showed
tenderness at the middle part of sacrum and the proximal parts of
femur.
A whole-body Tc-99m MDP bone scan (WBBS) and computed
tomography (CT) scanning of the abdomen/pelvis were requested
Anterior (A) and posterior (B) planes of
whole-body bone scan showing an
increased uptake at the greater trochanter
of the left femur (red cycle)
WaD4, WID5
Skeletal Structure and Function
Week 3 Day 4
Excessive alcohol intake results in the increased breakdown of ATP, resulting in the excessive
production of organic acids. These organic acids compete with urate for tubular secretion (urate
transporter involved in tubular secretion), resulting in hyperuricemia and a subsequent gout attack. The
same mechanism is observed in other conditions, such as lactic acidosis, diabetic ketoacidosis, and
salicylate toxicity.
5) Hyperuricemia is common in people treated with a loop or thiazide diuretic (usually at doses of >25
mg/day) and may contribute to new-onset gout or recurrence of established gout.
Diuretics reduce urate excretion by both direct and indirect mechanisms: by increasing urate reabsorption
by the proximal tubule, and by causing volume depletion which increases urate reabsorption by the
proximal tubule. Importantly, the effect is dose-dependent. If diuretic-induced gout occurs, it is usually
treated with a urate-lowering drug such as allopurinol, or using an alternative antihypertensive drug, such
as an angiotensin inhibitor or a dihydropyridine calcium channel blocker.
Other drugs causing hyperuricemia include aspirin, niacin, furosemide, and cyclosporine.
6) One of the first-line treatments of a gout flare is a nonsteroidal anti-inflammatory
drug (NSAID), such as indomethacin or naproxen, but glucocorticoids (either oral or
by intraarticular injection) are similar or superior in efficacy in most patients.
Low-dose colchicine is considered a second-line drug to treat an acute gout flare
because it has more adverse side effects and must be given within 36 hours of onset.
NSAIDs inhibit the cyclooxygenase enzymes, COX-1 and COX-2. Inhibition of these
enzymes downregulates the production of prostaglandins, which are important
mediators of inflammation. NSAIDs are also commonly used to treat fever by reduction
of prostaglandin E2 (PGE2) synthesis. Adverse effects of NSAIDs include
gastrointestinal problems (gastric ulcers and heartburn), kidney injury, and
cardiovascular events.
Allopurinol lowers serum urate levels, mostly by inhibiting xanthine oxidase. It helps
prevent recurrent gout flares and can treat tophaceous gout, but is not used to treat a gout
flare.
Allopurinol is a xanthine oxidase inhibitor (XOI) that irreversibly inhibits xanthine oxidase,
which normally converts hypoxanthine to xanthine and then to uric acid. Xanthine oxidase
converts allopurinol to alloxanthine (oxypurinol), which is an irreversible "suicide"
inhibitor of the enzyme. Thus, the pathway of the synthesis of uric acid is blocked,
decreasing serum uric acid levels.
Febuxostat is also an XOI but it has more side effects and a higher cost than allopurinol, so
it is only used as an alternative to allopurinol if that drug is not tolerated.
Probenecid enhances the elimination of uric acid but is not used as much as allopurinol
since it is less effective and is associated with multiple drug interactions.
A 62-year-old carpenter presents to your clinic complaining of worsening joint pain
in her hands. She states that the pain is present in all of her fingers, but is worse in
the right hand and gets better when she has a few days off from work. She denies
any paresthesias, fevers, or fatigue. Vital signs are normal. Physical exam reveals
hard, non-tender lesions overlying the proximal interphalangeal joints of the 2nd
and 3rd fingers of the right hand. All five digits of the right hand have a decreased
range of motion. An X-ray of her hands is shown.
Patients typically present with diffuse generalized muscle pain, red or brown urine, hyperkalemia,
pigmented urinary casts, and hypocalcemia. Urinalysis will characteristically be positive for blood
without the presence of red blood cells.
Oliguria occurs when urine production is reduced dramatically, defined as a decrease in urine output
to less than 500 mL per day or less than 0.5 ml/kg/hour. In contrast, anuria is the absence of urine
production, defined as a urine output of fewer than 100 milliliters per day.
Rhabdomyolysis is diagnosed by checking serum CK, normally contained within the
skeletal muscle. Rhabdomyolysis presents with serum CK levels at least 5 times the
upper limit of normal.
2) First and foremost, these patients need aggressive fluid resuscitation to improve
kidney perfusion and ameliorate kidney injury.
Skeletal Structure and Function
Week 3 Day 5
Musculoskeletal System
Congenital Disorders and Adverse Effects of Medications
Learning objectives:
1. Define and discuss Congenital Disorders and Adverse Effects of Medications on Musculoskeletal system
2. Review the steps involved in the evaluation Congenital Disorders and Adverse Effects of Medications on
Musculoskeletal system
3. Discuss the basic principles applied for management of Congenital Disorders and Adverse Effects of Medications
on Musculoskeletal system
A 1-year-old boy presents with his mother to the pediatrician for a routine checkup. On examination, the child is happy
and playful and meets normal cognitive development markers. However, the child’s arms and legs are not meeting
development goals, while his head and torso are. The mother states that the boy gets this from his father. Which of the
following mutations is most likely associated with this presentation?
Learning objective: Achondroplasia is the most common cause of dwarfism. It is due to a gain-of-function mutation in
the FGFR3 gene, which is inherited in an autosomal dominant pattern. The disease is characterized by small limbs and
an average-sized head and axial skeleton.
A 12-year-old boy develops muscle weakness, pain, vomiting, seizures, and severe headache. He also presents with
hemiparesis. A muscle biopsy shows "ragged red fibers." What is true about the mode of inheritance of the disease
described?
of mitochondrial myopathy known as MELAS syndrome. MELAS stands for mitochondrial encephalopathy, lactic
acidosis, and stroke-like symptoms. Those affected may also present with epilepsy, myopathy, recurrent headaches that
resemble migraines, hearing impairment, cardiac disease, diabetes, vision impairment, and short stature.
MELAS syndrome follows a mitochondrial inheritance; therefore, it is only transmitted through the mother. All
of her offspring will be affected, and all of her daughters will transmit the condition to future generations, but the sons
will not. The mitochondria have their own DNA (i.e., mtDNA). The most commonly affected gene in MELAS
syndrome is MT-TL1 (occurring in more than 80% of patients). A mutation affects the mitochondrial tRNA function,
leading to a disruption of the global process of intramitochondrial protein synthesis. The decreased protein synthesis
may ultimately lead to the observed decrease in the respiratory chain, triggering excessive lactic acid production.
The workup for MELAS syndrome consists of:
•Serum lactate and serum pyruvate
•mtDNA mutation studies
•Brain imaging studies
•Muscle biopsy
Management of MELAS syndrome is symptomatic and not specific, although CoQ10 and L-arginine have shown some
efficacy.
(a) Modified Gomori trichrome stain
showing several ragged red fibers
(arrowhead). (b) Cytochrome c oxidase
stain showing lightly stained Type-1 fibers
and Type II fibers, darker fibers, and a few
fibers with abnormal collections of
mitochondria (arrowhead). Note the
cytochrome c oxidase negative fibers,
which are usually seen in mitochondrial
encephalopathy, lactic acidosis, and stroke-
like episodes (MELAS). (c) Succinate
dehydrogenase staining showing a few
ragged blue fibers and intense staining in
the mitochondria of the blood vessels
(arrow). (d) Electron microscopy showing
an abnormal collection of mitochondria
with paracrystalline inclusions (arrowhead),
osmiophilic inclusions (large arrowhead),
and mitochondrial vacuoles (small
arrowhead).
Mode of inheritance Characteristics Examples
Autosomal dominant •Affected individuals in many Achondroplasia, familial
generations hypercholesterolemia,
•Male and females are equally Huntington disease, Marfan
affected syndrome
Autosomal recessive •Usually observed in only one Albinism, cystic fibrosis,
A.It is usually more severe in males. generation Kartagener syndrome,
•High risk in consanguineous phenylketonuria, sickle cell
B.It skips generations. families anemia
C.Mothers transmit it to 50% of their X-linked recessive •More severe in males (answer Fabry disease, Hemophilia A
A) and B, Lesch-Nyhan syndrome,
daughters and sons. •Sons of carrier mothers have a Wiskott-Aldrich syndrome,
50% chance of developing the Ornithine transcarbamylase
D.It is transmitted only through the disease deficiency
mother. •Skips generations
•Females must be homozygous
E.It can be transmitted through both to be affected
Learning objective: MELAS syndrome is a mitochondrial myopathy with a mitochondrial pattern of inheritance. The
mothers pass the mitochondria to each of their offspring. Their sons cannot transmit the disease, but their daughters will
transmit this disease to their future generations. The clinical hallmarks of MELAS include mitochondrial
encephalopathy (dementia), lactic acidosis, and stroke-like symptoms.
A 5-year-old boy is brought to the emergency department by his foster mother because of multiple injuries. She says
that he sustained these injuries while playing. Radiographic findings show multiple fractures in various stages of
healing. She notes that he has always bruised easily. His family history is unknown. His blood pressure is 118/78 mm
Hg, pulse is 76/min, and respirations are 15/min. Physical examination shows the findings in the image. What is the
most likely diagnosis in this patient?
A.Marfan syndrome
B.Wilson's disease
C.Osteogenesis imperfecta
D.Osteochondritis dissecans
E.Child abuse
Correct answer C: This patient's diagnosis is osteogenesis imperfecta (OI), or brittle bone disease. This disease is
caused by the defective synthesis of type 1 collagen. Since type 1 collagen is a major extracellular matrix component,
manifestations are seen in the bones, joints, and sclerae.
Too little bone formation results in brittle/fragile bones and recurrent fractures. Radiographs of the bones show multiple
injuries in different stages of healing. The choroidal veins are reflected through the thin sclera, giving the sclerae a
bluish appearance (see image below).
Learning objective: Osteogenesis imperfecta is due to a genetic defect in type 1 collagen, resulting in brittle bones,
hearing loss, discolored teeth, and blue sclerae.